Barron's SAT, 26th edition (2012)

Part 4. TEST YOURSELF

Chapter 10. Five Model SAT Tests

• 5 Model Tests

• Answer Keys

• Self-Evaluations

• Answer Explanations



You are now about to take a major step in preparing yourself to handle an actual SAT. Before you are 5 Model Tests patterned after current published SATs. Up to now, you’ve concentrated on specific areas and on general testing techniques. You’ve mastered tactics and answered practice questions. Now you have a chance to test yourself—thoroughly, repeatedly—before you walk in that test center door.

These 5 Model Tests resemble the actual SAT in format, in difficulty, and in content. When you take them, take them as if they were the actual SAT.

Please note: All directions reflect the actual exam, but for this eBook continue to record your answers separately.

BUILD YOUR STAMINA

Don’t start and then stop and take time out for a soda or for an important phone call. To do well on the SAT, you have to focus on the test, and nothing but the test, for hours at a time. Many high school students have never had to sit through a 4-hour examination before they take their first SAT. To survive such a long exam takes stamina, and, as marathon runners know, the only way to build stamina is to put in the necessary time during practice sessions.

REFINE YOUR SKILLS

You know how to maximize your score by tackling easy questions first and by eliminating wrong answers whenever you can. Put these skills into practice. If you find yourself spending too much time on any one question, make an educated guess and move on. Remember to check frequently to make sure you are indicating your answers in the right spots. This is a great chance for you to get these skills down pat.

SPOT YOUR WEAK POINTS

Do you need a bit more drill in a particular area? After you take each test, consult the self-evaluation section and the answer explanations for that test to pinpoint any areas that need work. Don’t just evaluate your scores. Build your skills. Read the answer explanations for each question you answered incorrectly, each question you omitted, and each question you answered correctly but found hard. The answer explanation section is tailor-made to help you. You’ll find reminders of tactics, definitions of terms, explanations of why the correct answer works. You’ll even find an occasional shortcut or two and an explanation of why an incorrect answer didn’t work.

Use the answer explanation section to help you spot specific types of questions that you want to review. Suppose, for example, you’ve omitted answering several reading questions on a test. Going through the answer explanations, you find that all of these questions are of the Inference type. You know right then that you can boost your score by mastering the specific skill needed to deal with inferences.

TAKE A DEEP BREATH—AND SMILE!

It’s hard to stay calm when those around you are tense, and you’re bound to run into some pretty tense people when you take the SAT. (Not everyone works through this book, unfortunately.) So you may experience a slight case of “exam nerves” on the big day. Don’t worry about it.

  1. Being keyed up for an examination isn’t always bad: you may outdo yourself because you are so worked up.

  2. Total panic is unlikely to set in: you know too much.

You know you can handle a 123-hour test.

You know you can handle the sorts of questions you’ll find on the SAT.

You know you can omit several questions and still score high. Answer correctly only 50–60% of the questions, omitting others, and you’ll get a better than average score (and hundreds of solid, wellknown colleges are out there right now, looking for serious students with just that kind of score).

Answer more than that correctly and you should wind up with a superior score.

MAKE YOUR PRACTICE PAY—APPROXIMATE THE TEST

  1. Whenever possible, complete an entire Model Test at one sitting.

  2. Use a clock or timer.

  3. Allow precisely 25 minutes for each of Sections 1 through 7, 20 minutes for each of Sections 8 and 9, and 10 minutes for Section 10. (If you finish any section in less than the allotted time, review your answers or go back to a question you omitted.)

  4. After each section, give yourself a 1-minute break, and take 10-minute breaks after Sections 3 and 7.

  5. Allow no talking in the test room.

  6. Work rapidly without wasting time.

Model Test 1

Answer Sheet—Test 1

Section 1                ESSAY

img

If a section has fewer questions than answer spaces, leave the extra spaces blank.

Section 2

img

Section 3

img

Section 4

img

Section 6

img

Section 7

img

img

Section 8

img

Section 9

img

Section 10

img

Test 1

SECTION 1

Time—25 Minutes

ESSAY

The excerpt appearing below makes a point about a particular topic. Read the passage carefully, and think about the assignment that follows.

The novelist John Hersey wrote, “Learning starts with failure;
the first failure is the beginning of education.”

ASSIGNMENT: What are your thoughts on the idea that failure is necessary for education to take place? Compose an essay in which you express your views on this topic. Your essay may support, refute, or qualify the views expressed in the excerpt. What you write, however, must be relevant to the topic under discussion. Additionally, you must support your viewpoint, indicating your reasoning and providing examples based on your studies and/or experience.

SECTION 2

Time—25 Minutes 24 Questions

Select the best answer to each of the following questions; then blacken the appropriate space on your answer sheet.

Each of the following sentences contains one or two blanks; each blank indicates that a word or set of words has been left out. Below the sentence are five words or phrases, lettered A through E. Select the word or set of words that best completes the sentence.

Example:

Fame is ----; today’s rising star is all too soon tomorrow’s washed-up has-been.

(A) rewarding

(B) gradual

(C) essential

(D) spontaneous

(E) transitory

123

  1. Although he is ---- about the problems that still confront blacks in ballet, Mitchell nevertheless is optimistic about the future, especially that of his own dance company.

(A) hopeful

(B) uninformed

(C) abstract

(D) realistic

(E) unconcerned

  2. Despite all its ----, a term of enlistment in the Peace Corps can be both stirring and satisfying to a college graduate still undecided on a career.

(A) rewards

(B) incongruity

(C) prestige

(D) seclusion

(E) frustrations

  3. Although he had numerous films to his credit and a reputation for technical ----, the moviemaker lacked originality; all his films were sadly ---- of the work of others.

(A) skill..independent

(B) ability..unconscious

(C) expertise..derivative

(D) competence..contradictory

(E) blunders..enamored

  4. John Gielgud crowned a distinguished career of playing Shakespearean roles by giving a performance that was ----.

(A) mediocre

(B) outmoded

(C) superficial

(D) unsurpassable

(E) insipid

  5. Those interested in learning more about how genetics applies to trees will have to ---- the excellent technical journals where most of the pertinent material is ---- .

(A) subscribe to..ignored

(B) suffer through..located

(C) rely on..unrepresented

(D) resort to..found

(E) see through..published

  6. Rent control restrictions on small apartment owners may unfortunately ---- rather than alleviate housing problems.

(A) resolve

(B) diminish

(C) castigate

(D) minimize

(E) exacerbate

  7. In the light of Dickens’s description of the lively, even ---- dance parties of his time, Sharp’s approach to country dancing may seem too formal, suggesting more ---- than is necessary.

(A) sophisticated..expertise

(B) rowdy..decorum

(C) prudish..propriety

(D) lewd..ribaldry

(E) enjoyable..vitality

  8. The heretofore peaceful natives, seeking ---- the treachery of their supposed allies, became, ---- according to their perspective, embittered and vindictive.

(A) acquiescence in..understandably

(B) magnanimity towards..logically

(C) evidence of..impartially

(D) retribution for..justifiably

(E) exoneration of..ironically

Read each of the passages below, and then answer the questions that follow the passage. The correct response may be stated outright or merely suggested in the passage.

Questions 9 and 10 are based on the following passage.

On the playgrounds of Brooklyn, basketball is
more religious rite than sport. Its devotees are on
the court ten hours a day, six days a week.
Seventeen- and eighteen-year-olds have rheumatoid

Line (5) knees from the constant pounding of their
feet on the asphalt. They play through the afternoon
heat with little more to fuel them than a can
of soda, and they play at night in the dim illumination
of nearby streetlights. They play even in

(10) the dead of winter, banging away at the netless
rims, hoping for salvation in the form of a contract
with the NBA.

  9. The facilities for playing basketball available to the seventeen- and eighteen-year-olds described in the passage can best be characterized as

(A) professional

(B) sheltered

(C) rudimentary

(D) well designed

(E) seldom accessible

10. The “salvation” mentioned in the final sentence most likely refers to

(A) a realistic expectation of athletic success

(B) the potential for excellence that exists in all players

(C) formal promises made to amateur athletes by the NBA

(D) the ideal of sportsmanship exemplified by professional athletes

(E) a deliverance from poverty through professional sports

Questions 11 and 12 are based on the following passage.

This excerpt from Mark Twain’s Roughing It describes an animal Twain encountered during his travels in the West.

The coyote is a long, slim, sick and sorry-looking
skeleton, with a gray wolf-skin stretched over
it, a tolerably bushy tail that forever sags down, a
furtive and evil eye, and a long, sharp face, with

Line (5) slightly lifted lip and exposed teeth. He has a general
slinking expression all over. The coyote is a
living, breathing allegory of Want. He is always
hungry. He is always poor, out of luck, and friendless.
The meanest creatures despise him, and even

(10) the fleas would desert him for a velocipede.

11. The passage above can best be characterized as an example of

(A) scientific analysis

(B) nostalgic anecdote

(C) humorous exaggeration

(D) objective reportage

(E) lyrical description

12. In line 9, “meanest” most nearly means

(A) most ordinary

(B) most stingy

(C) most ashamed

(D) most effective

(E) most contemptible

Questions 13–24 are based on the following passage.

The following excerpt is taken from “Life on the Rocks: the Galapagos” by writer Annie Dillard. Like Charles Darwin, originator of the theory of evolution, Dillard visited the Galapagos Islands in the Pacific. In this passage she muses on the islands, on Darwin, and on the evolutionary process.

Charles Darwin came to the Galapagos in
1835, on the Beagle; he was twenty-six. He threw
the marine iguanas as far as he could into the
water; he rode the tortoises and sampled their

Line (5) meat. He noticed that the tortoises’ carapaces varied
wildly from island to island; so also did the
forms of various mockingbirds. He made collections.
Nine years later he wrote in a letter, “I am
almost convinced (quite contrary to the opinion I

(10) started with) that species are not (it is like
confessing a murder) immutable.” In 1859 he published
On the Origin of Species, and in 1871
The Descent of Man. It is fashionable now to disparage
Darwin’s originality; not even the surliest of

(15) his detractors, however, faults his painstaking
methods or denies his impact.

It all began in the Galapagos, with these
finches. The finches in the Galapagos are called
Darwin’s finches; they are everywhere in the

(20) islands, sparrowlike, and almost identical but for
their differing beaks. At first Darwin scarcely
noticed their importance. But by 1839, when he
revised his journal of the Beagle voyage, he
added a key sentence about the finches’ beaks:

(25) “Seeing this gradation and diversity of structure
in one small, intimately related group of birds,
one might really fancy that from an original
paucity of birds in this archipelago, one species
had been taken and modified for different ends.”

(30) And so it was.

The finches come when called. I don’t know
why it works, but it does. Scientists in the
Galapagos have passed down the call: you say
psssssh psssssh psssssh psssssh until you run out

(35) of breath; then you say it again until the island
runs out of birds. You stand on a flat of sand by a
shallow lagoon rimmed in mangrove thickets and
call the birds right out of the sky. It works anywhere,
from island to island.

(40)      Once, on the island of James, I was standing
propped against a leafless palo santo tree on a
semiarid inland slope, when the naturalist called
the birds.

From other leafless palo santo trees flew the

(45) yellow warblers, speckling the air with bright
bounced sun. Gray mockingbirds came running.
And from the green prickly pear cactus, from the
thorny acacias, sere grasses, bracken and manzanilla,
from the loose black lava, the bare dust,

(50) the fern-hung mouths of caverns or the tops of
sunlit logs—came the finches. They fell in from
every direction like colored bits in a turning
kaleidoscope. They circled and homed to a vortex, like
a whirlwind of chips, like draining water. The tree

(55) on which I leaned was the vortex. A dry series of
puffs hit my cheeks. Then a rough pulse from the
tree’s thin trunk met my palm and rang up my
arm—and another, and another. The tree trunk
agitated against my hand like a captured cricket: I

(60) looked up. The lighting birds were rocking the
tree. It was an appearing act: before there were
barren branches; now there were birds like leaves.

Darwin’s finches are not brightly colored; they
are black, gray, brown, or faintly olive. Their

(65) names are even duller: the large ground finch, the
medium ground finch, the small ground finch; the
large insectivorous tree finch; the vegetarian tree
finch; the cactus ground finch, and so forth. But
the beaks are interesting, and the beaks’ origins

(70) even more so.

Some finches wield chunky parrot beaks modified
for cracking seeds. Some have slender warbler
beaks, short for nabbing insects, long for
probing plants. One sports the long chisel beak of

(75) a woodpecker; it bores wood for insect grubs and
often uses a twig or cactus spine as a pickle fork
when the grub won’t dislodge. They have all
evolved, fanwise, from one bird.

The finches evolved in isolation. So did everything

(80) else on earth. With the finches, you can see
how it happened. The Galapagos islands are near
enough to the mainland that some strays could
hazard there; they are far enough away that those
strays could evolve in isolation from parent

(85) species. And the separate islands are near enough
to each other for further dispersal, further isolation,
and the eventual reassembling of distinct
species. (In other words, finches blew to the
Galapagos, blew to various islands, evolved into

(90) differing species, and blew back together again.)
The tree finches and the ground finches, the
woodpecker finch and the warbler finch, veered
into being on isolated rocks. The witless green
sea shaped those beaks as surely as it shaped the

(95) beaches. Now on the finches in the palo santo
tree you see adaptive radiation’s results, a fluorescent
spray of horn. It is as though an archipelago
were an arpeggio, a rapid series of distinct but
related notes. If the Galapagos had been one unified

(100) island, there would be one dull note, one
super-dull finch.

13. Dillard’s initial portrayal of Darwin (lines 1–5) conveys primarily a sense of his

(A) methodical research

(B) instant commitment

(C) youthful playfulness

(D) lack of original thought

(E) steadiness of purpose

14. From lines 8–11 one can conclude that Darwin originally viewed species as

(A) unchanging

(B) original

(C) ambiguous

(D) evolutionary

(E) indistinguishable

15. In the phrase “It all began in the Galapagos” (line 17), “It” refers to the origins of

(A) sentient life

(B) distinct species of creatures

(C) Darwin’s theory of evolution

(D) controlled experimentation

(E) Darwin’s interest in nature

16. In line 29, “ends” most nearly means

(A) borders

(B) extremities

(C) limits

(D) purposes

(E) deaths

17. The use of the phrase “run out” two times (lines 32–36) emphasizes the

(A) waste of energy involved

(B) difference between the actions of humans and birds

(C) impatience of the naturalists calling the birds

(D) nervousness of the author in strange situations

(E) overwhelming response of the birds

18. In line 60, “lighting” most nearly means

(A) illuminating

(B) landing

(C) shining

(D) weightless

(E) flapping

19. The pulse that Dillard feels (lines 56–58) is most likely

(A) the agitated beating of her heart

(B) the rhythm of the birds’ touching down

(C) the leaping of crickets against the tree

(D) a painful throbbing in her arm

(E) the wind of the birds’ passing

20. Dillard’s description of the finches (lines 71–75) serves chiefly to

(A) contrast their overall drabness with their variety in one specific aspect

(B) illustrate the predominance of tree finches over ground finches

(C) emphasize the use of memorable names to distinguish different species

(D) convey a sense of the possibilities for further evolution in the finch family

(E) distinguish them from the warblers and mockingbirds found in the islands

21. Lines 71–78 suggest that the finches’ beaks evolved in ways that

(A) mimicked a fanlike shape

(B) protected the birds from attack

(C) captured Darwin’s interest

(D) enhanced the birds’ attractiveness

(E) enabled them to reach nourishment

22. In line 83, “hazard” most nearly means

(A) venture

(B) speculate

(C) be imperiled

(D) run aground

(E) develop

23. The “fluorescent spray of horn” referred to by the author in lines 96 and 97 is most likely

(A) a series of musical notes

(B) a flock of birds

(C) the birds’ shiny beaks

(D) branches of the palo santo tree

(E) a primitive musical instrument

24. In the final paragraph, the author does all of the following EXCEPT

(A) restate an assertion

(B) make a comparison

(C) define a term

(D) refute an argument

(E) describe a sequence of events

S T O P

YOU MAY GO BACK AND REVIEW THIS SECTION IN THE REMAINING TIME, BUT DO NOT WORK IN ANY OTHER SECTION UNTIL TOLD TO DO SO.

SECTION 3

Time—25 Minutes 20 Questions

For each problem in this section determine which of the five choices is correct and blacken the corresponding choice on your answer sheet. You may use any blank space on the page for your work.

Notes:

• You may use a calculator whenever you think it will be helpful.

• Only real numbers are used. No question or answer on this test involves a complex or imaginary number.

• Use the diagrams provided to help you solve the problems. Unless you see the words “Note: Figure not drawn to scale” under a diagram, it has been drawn as accurately as possible. Unless it is stated that a figure is three-dimensional, you may assume it lies in a plane.

• For any function f, the domain, unless specifically restricted, is the set of all real numbers for which f (x) is also a real number.

Reference Information

123

123

  1. If 5c + 3 = 3c + 5, what is the value of c?

(A) –1

(B) 0

(C) 1

(D) 3

(E) 5

123

  2. In the figure above, C is the only point that right triangle ABC and square CDEF have in common. What is the value of a + b?

(A) 135

(B) 180

(C) 210

(D) 225

(E) 270

  3. A lacrosse team raised some money. The members used 74% of the money to buy uniforms, 18% for equipment, and the remaining $216 for a team party. How much money did they raise?

(A) $2400

(B) $2450

(C) $2500

(D) $2600

(E) $2700

  4. For all positive numbers a and b, let 123 If n > 1, what does n 123 123 equal?

(A) 123

(B) 123

(C) 123

(D) 123

(E) 1

123

Note: Figure not drawn to scale

  5. In the figure above, what is the value of b?

(A) 30

(B) 36

(C) 45

(D) 60

(E) 72

123

Note: Figure not drawn to scale

  6. In the figure above, if x is 150 more than y, what is the value of y?

(A) 10

(B) 15

(C) 20

(D) 25

(E) 30

  7. Heidi wrote the number 1 on 1 slip of paper, the number 2 on 2 slips of paper, the number 3 on 3 slips of paper, the number 4 on 4 slips of paper, the number 5 on 5 slips of paper, and the number 6 on 6 slips of paper. All the slips of paper were placed in a bag, and Sally drew 1 slip at random. What is the probability that the number on the slip Sally drew was odd?

(A) 123

(B) 123

(C) 123

(D) 123

(E) 123

  8. For how many positive numbers a is it true that a × a × a = a + a + a?

(A) 0

(B) 1

(C) 2

(D) 3

(E) more than 3

  9. Last year Jose sold a painting for $2000. If he made a 25% profit on the sale, how much had he paid for the painting?

(A) $1200

(B) $1500

(C) $1600

(D) $2400

(E) $2500

10. For any positive integer n > 1, n! represents the product of the first n positive integers. For example, 3! = 1 × 2 × 3 = 6. Which of the following is (are) equal to 123

  I. 5! – 4! – 3!

 II. 123

III. 15(3!)

(A) I only

(B) II only

(C) III only

(D) I and III only

(E) I, II, and III

11. A rectangle is twice as long as it is wide. If the width is a, what is the length of a diagonal?

(A) a 123

(B) a 123

(C) a 123

(D) 3a

(E) 5a

12. If abc = 1, which of the following could be the number of integers among ab, and c?

  I. 1

 II. 2

III. 3

(A) None

(B) I only

(C) I and II only

(D) I and III only

(E) I, II, and III

13. At Essex High School 100 students are taking chemistry and 80 students are taking biology. If 20 students are taking both chemistry and biology, what is the ratio of the number of students taking only chemistry to the number taking only biology?

(A) 123

(B) 123

(C) 123

(D) 123

(E) 123

123

14. In the figure above, a small square is drawn inside a large square. If the shaded area and the white area are equal, what is the ratio of the side of the large square to the side of the small square?

(A) 123

(B) 123

(C) 123

(D) 123

(E) It cannot be determined from the information given.

123

15. In rectangle ABCD above, diagonal AC makes a 30° angle with side AD. If AC = 10, what is the area of the rectangle?

(A)   25 123

(B)   25 123

(C)   48

(D)   50

(E) 100

16. The value of 10 pounds of gold is d dollars, and a pound of gold has the same value as p pounds of silver. What is the value, in dollars, of one pound of silver?

(A) 123

(B) 123

(C) 123

(D) 123

(E) 123

123

17. In the figure above, container I is a rectangular solid whose base is a square 4 inches on a side, and container II is a cylinder whose base is a circle of diameter 4 inches. The height of each container is 5 inches. How much more water, in cubic inches, will container I hold than container II?

(A)   4(4 – 123)

(B) 20(4 – 123)

(C) 80(123 – 1)

(D) 80(1 – 123)

(E) It cannot be determined from the information given.

18. The number of cells growing in a particular Petri dish doubles every 30 minutes. If at 8:00 A.M. there were 60 cells in the dish, how many were there at noon of the same day?

(A) 60 × 2–8

(B) 60 × 2–4

(C) 60 × 24

(D) 60 × 44

(E) 60 × 48

19. The distance between Ali’s house and college is 23 exactly 135 miles. If she drove 123 of the distance in 135 minutes, what was her average speed, in miles per hour?

(A) 40

(B) 45

(C) 60

(D) 67.5

(E) 75

20. The average (arithmetic mean) weight of five students is 150.4 pounds. If no student weighs less than 130 pounds and if the difference in the weights of any two students is at least 5 pounds, what is the most, in pounds, that any one of the students can weigh?

(A) 172

(B) 192

(C) 202

(D) 232

(E) 242

S T O P

YOU MAY GO BACK AND REVIEW THIS SECTION IN THE REMAINING TIME, BUT DO NOT WORK IN ANY OTHER SECTION UNTIL TOLD TO DO SO.

SECTION 4

Time—25 Minutes 35 Questions

Select the best answer to each of the following questions; then blacken the appropriate space on your answer sheet.

Some or all parts of the following sentences are underlined. The first answer choice, (A), simply repeats the underlined part of the sentence. The other four choices present four alternative ways to phrase the underlined part. Select the answer that produces the most effective sentence, one that is clear and exact, and blacken the appropriate space on your answer sheet. In selecting your choice, be sure that it is standard written English, and that it expresses the meaning of the original sentence.

Example:

The first biography of author Eudora Welty came out in 1998 and she was 89 years old at the time.

(A) and she was 89 years old at the time

(B) at the time when she was 89

(C) upon becoming an 89 year old

(D) when she was 89

(E) at the age of 89 years old

123

  1. Complaining that he couldn’t hear hardly anything, he asked Dr. Brown, the otologist, whether he should get a hearing aid.

(A) Complaining that he couldn’t hear hardly anything,

(B) Complaining that he couldn’t hardly hear anything,

(C) He complained that he couldn’t hear hardly anything,

(D) Complaining that he could hear hardly anything,

(E) Because he couldn’t hear hardly anything,

  2. Shakespeare wrote many plays, they are now being presented on public television.

(A) Shakespeare wrote many plays, they are now being presented on public television.

(B) Shakespeare wrote many plays, and they are now being presented on public television.

(C) Shakespeare wrote many plays, public television is now presenting them.

(D) The many plays Shakespeare wrote that are now being presented on public television.

(E) Shakespeare wrote many plays that are now being presented on public television.

  3. Many alcoholics attempt to conceal their problem from their fellow workers, but invariably failing to keep their secret.

(A) but invariably failing to keep their secret

(B) but they invariably fail to keep their secret

(C) but fail, invariably, to keep their secret

(D) who invariably fail to keep their secret

(E) who they invariably fail to keep their secret from

  4. Upon considering the facts of the case, the solution was obvious; consequently, Holmes sent for the police.

(A) Upon considering

(B) When considering

(C) Considering

(D) In consideration of

(E) When he considered

  5. Familiar with the terrain from previous visits, the explorer’s search for the abandoned mine site was a success.

(A) the explorer’s search for the abandoned mine site was a success

(B) the success of the explorer’s search for the abandoned mine site was assured

(C) the explorer succeeded in finding the abandoned mine site

(D) the search by the explorer for the abandoned mine site was successful

(E) the explorer in his search for the abandoned mine site was a success

  6. Economic conditions demand not only cutting wages and prices but also to reduce inflation-raised tax rates.

(A) not only cutting wages and prices but also to reduce

(B) we not only cut wages and prices but also reduce

(C) to not only cut wages and prices but also to reduce

(D) not only to cut wages and prices but also to reduce

(E) not only a cut in wages and prices but also to reduce

  7. He interviewed several candidates who he thought had the experience and qualifications he required.

(A) who he thought

(B) whom he thought

(C) of whom he thought

(D) he thought who

(E) which he thought

  8. It is typical of military service for a skilled technician to be inducted and then you spend your whole tour of duty peeling potatoes and cleaning latrines.

(A) then you spend your whole tour of duty

(B) to spend your whole tour of duty

(C) then they spend their whole tour of duty

(D) to spend their whole tour of duty

(E) then spend her whole tour of duty

  9. In years past, teenagers typically passed notes to their friends in class rather than electronic instant messages today.

(A) class rather than electronic instant messages today

(B) class, but today it is electronic instant messages

(C) class; today they send electronic instant messages

(D) class instead of electronic instant messages today

(E) class; instead, teenagers today sending instant messages electronically

10. George Balanchine’s inspiration has had a great effect on many later choreographers who came after him, including Danish-born Peter Martins.

(A) George Balanchine’s inspiration has had a great effect on many later choreographers who came after him

(B) George Balanchine’s inspiration has greatly effected many later choreographers who came after him

(C) The inspiration of George Balanchine was great for many later choreographers who came after him

(D) Many choreographers who came after him have been affected greatly by the inspiration of George Balanchine

(E) George Balanchine has inspired many later choreographers

11. According to Freud, the aim of psychotherapy is to trace neurotic symptoms back to their unconscious roots and expose these roots to mature, rational judgment, thereby depriving them of their compulsive power.

(A) judgment, thereby depriving them of their compulsive power

(B) judgment; and thereby it deprives them of their compulsive power

(C) judgment; thereby depriving them of their compulsive power

(D) judgment, thereby it deprives them of their compulsive power

(E) judgment, thereby it deprives them of its compulsive power

The sentences in this section may contain errors in grammar, usage, choice of words, or idioms. Either there is just one error in a sentence or the sentence is correct. Some words or phrases are underlined and lettered; everything else in the sentence is correct.

If an underlined word or phrase is incorrect, choose that letter; if the sentence is correct, select No error. Then blacken the appropriate space on your answer sheet.

Example:

The region has a climate 123 plants 123 rarely 123 more than twelve inches 123

123

12. I 123 your tale of military intrigue; the sophisticated secret weapons and the 123 actions 123 were exhibited by just one man 123 incredible. 123

13. The animals 123 were chosen 123 Democratic and Republican parties, the donkey and the elephant, 123 by the 123 Thomas Nast. 123

14. I 123 you and 123to supply the 123 for the annual statement that must be prepared 123 the spring meeting. 123

15. 123 of the space shuttle Challenger explosion, 123 seven crew members 123 the NASA program underwent a 123 of priorities. 123

16. Twenty-five 123 123 paper clips, were drawing on the blackboard, and 123 one another 123 teacher went searching for milk and cookies. 123

17. Recent medical breakthroughs, including the 123 a vaccine to slow the AIDS virus, 123 researchers; 123 a cure is still 123 them. 123

18. 123 the producer took the musical to Broadway, he 123 the show with all 123 actors and actresses booked in summer stock theaters for last-123

19. Neither the midlife career change applicant 123 the young, inexperienced applicant 123 it easy 123 a career in data processing 123 a shortage of job openings. 123

20. Even 123 you have endured a cold winter in subzero weather, 123 possible 123 to tropical temperatures in the summer. 123

21. 123 you buy a condominium, you will have 123 owning a house entails, but you 123 the 123 rewards. 123

22. We have 123 that we can end hostilities in 123 of the world by providing food to both sides, bringing the opposing forces to the 123 and 123 financial aid to both sides once peace is established. 123

23. Numerous 123 short stories include works by Isaac Bashevis Singer who, 123 in the United States for more than fifty years, 123 primarily in Yiddish.123

24. Public television 123 in raising money for 123 future programs through marathon 123

25. 123 the bank guard closed the doors, a riot 123 the long lines and 123 tellers. 123

26. The 123 that 123 the sun 123 Earth 123 Copernicus in the sixteenth century. 123

27. The opera company members, 123 123 manager Joseph Volpe to conductor James Levine, joined forces 123 to 123 tenor Luciano Pavarotti. 123

28. Both major high school debate teams—123 eager 123 this year’s National Forensics League competition—123 to review 123 the videos of last year’s tournament. 123

29. Improvements in the global positioning system (GPS) 123 pilots using the system to guide aircraft 123 to the runway 123 severe weather creates 123 zero visibility. 123

The passage below is the unedited draft of a student’s essay. Parts of the essay need to be rewritten to make the meaning clearer and more precise. Read the essay carefully.

The essay is followed by six questions about changes that might improve all or part of the organization, development, sentence structure, use of language, appropriateness to the audience, or use of standard written English. In each case, choose the answer that most clearly and effectively expresses the student’s intended meaning. Indicate your choice by blackening the corresponding space on the answer sheet.

[1] It is difficult to deny that the world of music has changed greatly in the past thirty years. [2] The style, sound, technology, and lyrics of music have been altered greatly. [3] In the last three decades, several new categories of music have come into being.

[4] One reason why music has changed so greatly is that artists use music as a tool to publicize certain social messages. [5] Although many artists of the 1970s used this method as well, their issues were not as severe that banning their album was possible. [6] For example, one rap-singer, Ice-T, used his album to promote “cop-killing.” [7] The idea was so offensive that many believed the album should be banned. [8] The controversy caused by Ice-T made the Arista record company refuse to continue production of the album.

[9] Another way in which music has changed is lyrics.

[10] When you listen to certain heavy metal or rap groups, one may notice foul and obscene language used. [11] Some of the references to sex are shocking. [12] In past eras, such language in recorded music was unheard of.

[13] Technological changes in music have occurred.

[14] With the advent of highly advanced musical devices and many digital effects, the sounds of music have been completely altered. [15] Rock and roll was invented in early 1950s. [16] When you listen to heavy metal, you hear more distorted guitar sounds than in music of the 60s and 70s. [17] In the era of electronic instruments, the variety of possible sounds is incredible. [18] Present day sounds could never have been achieved in previous years because the technology was not at hand. [19] New music utilizes electronically produced sounds never heard before. [20] Computers generate everything from the human voice under water to the sound of whales. [21] There are no limits to what the music of the future will sound like.

30. Which of the following is the best revision of the underlined segment of sentence 5 below?

Although many artists of the 1970s used this method as well, their issues were not as severe that banning their album was possible.

(A) the issues were less severe than those which caused banning their album to be possible.

(B) their issues were not as severe that their albums were in danger of being banned.

(C) they never raised issues that could have caused their albums to be banned.

(D) the issues they raised were not serious enough that banning their album was a possibility.

(E) they raised less serious issues and banning their albums was not likely.

31. In view of the sentences that precede and follow sentence 10, which is the most effective revision of sentence 10?

(A) Listening to certain heavy metal or rap groups, lyrics containing obscenities are often heard.

(B) Obscene language is common in the songs of heavy metal and rap groups.

(C) Certain heavy metal and rap groups use foul and obscene language.

(D) Obscenities are often heard when one listens to the lyrics of certain heavy metal or rap groups.

(E) Listening to obscene language and listening to the lyrics of certain heavy metal and rap groups.

32. In the context of the entire essay, which revision of sentence 13 provides the most effective transition between paragraphs 3 and 4?

(A) Technological changes in music also have occurred.

(B) Also, technology has changed musical sounds.

(C) Noticeable changes in music’s sounds have come about through technological changes.

(D) Changes in musical technology has changed musical sound, too.

(E) But the most noticeable change in music has been its sound.

33. In a revision of the entire essay, which of the following sentences most needs further development?

(A) Sentence 3

(B) Sentence 7

(C) Sentence 8

(D) Sentence 19

(E) Sentence 20

34. Which of the following sentences should be deleted to improve the unity and coherence of paragraph 4?

(A) Sentence 14

(B) Sentence 15

(C) Sentence 16

(D) Sentence 17

(E) Sentence 18

35. With regard to the organization of the entire essay, which is the best revision of sentence 2 in the introductory paragraph?

(A) In the past thirty years, not only the style, sound, and technology has changed, but the lyrics have, too.

(B) Having undergone a change in the style, sound, and technology, musical lyrics have altered also.

(C) Changes in musical sound have occurred, while the technology and lyrics have tremendously altered the style of music.

(D) Musicians have transformed today’s music in style and sound, creating new lyrics and using new technology.

(E) Along with changes in sound and technology, the lyrics of music have changed, too.

S T O P

YOU MAY GO BACK AND REVIEW THIS SECTION IN THE REMAINING TIME, BUT DO NOT WORK IN ANY OTHER SECTION UNTIL TOLD TO DO SO.

SECTION 6

Time—25 Minutes 24 Questions

Select the best answer to each of the following questions; then blacken the appropriate space on your answer sheet.

Each of the following sentences contains one or two blanks; each blank indicates that a word or set of words has been left out. Below the sentence are five words or phrases, lettered A through E. Select the word or set of words that best completes the sentence.

Example:

Fame is ----; today’s rising star is all too soon tomorrow’s washed-up has-been.

(A) rewarding

(B) gradual

(C) essential

(D) spontaneous

(E) transitory

123

  1. Because our supply of fossil fuel has been sadly ----, we must find ---- sources of energy.

(A) stored..hoarded

(B) compensated..significant

(C) exhausted..inefficient

(D) increased..available

(E) depleted..alternative

  2. He is much too ---- in his writings: he writes a page when a sentence should suffice.

(A) devious

(B) lucid

(C) verbose

(D) efficient

(E) pleasant

  3. The abundance and diversity of insects is the cumulative effect of an extraordinarily low ---- rate: bugs endure.

(A) metabolic

(B) density

(C) extinction

(D) percentage

(E) standard

  4. Pre-Spanish art in Mexico is not a ---- art; they are mistaken who see in its bold simplifications or wayward conceptions an inability to ---- technical difficulties.

(A) formal..ignore

(B) graphic..understand

(C) primitive..nurture

(D) crude..overcome

(E) revolutionary..instigate

  5. Are we to turn into spineless ----, afraid to take a ---- stand, unable to answer a question without pussyfooting?

(A) disciples..positive

(B) hedonists..compromising

(C) criminals..defiant

(D) critics..constructive

(E) equivocators..forthright

Read the passages below, and then answer the questions that follow them. The correct response may be stated outright or merely suggested in the passages.

Questions 6–9 are based on the following passages.

Both passages relate to the career of the abolitionist Frederick Douglass. Passage 1 comes from the introduction to a collection of his short prose. Passage 2 is excerpted from Douglass’s letter to his former master, written while Douglass was in England.

Passage 1

To elude slave catchers, the fugitive slave
Frederick Baily changed his name, becoming
Frederick Douglass, abolitionist spokesman and
author. When he published his autobiography,

Line (5) however, Douglass exposed himself to recapture:
federal laws gave Douglass’s ex-master the right
to seize his property. Douglass traveled to Britain,
where slavery was illegal; there he worked to gain
support for America’s anti-slavery movement.

(10) After two years, British friends unexpectedly
bought his freedom, allowing him to return home
to continue the fight. Some abolitionists criticized
Douglass, however, saying that by letting his freedom
be bought he acknowledged his master’s

(15) right to own him.

Passage 2

I have often thought I should like to explain to
you the grounds upon which I have justified
myself in running away from you…. We are distinct
persons, and are each equally provided with

(20) faculties necessary to our individual existence. In
leaving you, I took nothing but what belonged to
me, and in no way lessened your means for
obtaining an honest living…. I therefore see no
wrong in any part of the transaction. It is true, I

(25) went off secretly; but that was more your fault
than mine. Had I let you into the secret, you
would have defeated the enterprise entirely; but
for this, I should have been really glad to have
made you acquainted with my intentions to leave.

  6. In line 7, “property” most nearly means

(A) parcel of land

(B) right of ownership

(C) characteristic trait

(D) personal possession

(E) particular virtue

  7. As described in the final sentence of Passage 1, the attitude of some abolitionists to the purchase of Douglass’s freedom can best be characterized as

(A) enthusiastic

(B) indifferent

(C) negative

(D) envious

(E) sympathetic

  8. Compared to Passage 2, Passage 1 can be described as

(A) figurative rather than literal

(B) expository rather than argumentative

(C) rhetorical rather than unembellished

(D) descriptive rather than factual

(E) subjective rather than objective

  9. The “enterprise” to which Douglass refers in the final sentence of Passage 2 is

(A) a financial transaction

(B) the letter to his former master

(C) his escape from slavery

(D) his return from England

(E) the means of earning an honest living

Questions 10–15 are based on the following passage.

The following passage is taken from a major historical text on life in the Middle Ages.

To the world when it was half a thousand
years younger, the outlines of all things seemed
more clearly marked than to us. The contrast
between suffering and joy, between adversity and

Line (5) happiness, appeared more striking. All experience
had yet to the minds of men the directness and
absoluteness of the pleasure and pain of child-life.
Every event, every action, was still embodied in
expressive and solemn forms, which raised them

(10) to the dignity of a ritual. For it was not merely the
great facts of birth, marriage, and death which, by
their sacredness, were raised to the rank of mysteries;
incidents of less importance, like a journey,
a task, a visit, were equally attended by a thousand

(15) formalities: benedictions, ceremonies,
formulae.

Calamities and indigence were more afflicting
than at present; it was more difficult to guard
against them, and to find solace. Illness and

(20) health presented a more striking contrast; the cold
and darkness of winter were more real evils.
Honors and riches were relished with greater
avidity and contrasted more vividly with surrounding
misery. We, at the present day, can

(25) hardly understand the keenness with which a fur
coat, a good fire on the hearth, a soft bed, a glass
of wine, were formerly enjoyed.

Then, again, all things in life were of a proud
or cruel publicity. Lepers sounded their rattles

(30) and went about in processions, beggars exhibited
their deformity and their misery in churches.
Every order and estate, every rank and profession,
was distinguished by its costume. The great lords
never moved about without a glorious display of

(35) arms and liveries, exciting fear and envy. Executions
and other public acts of justice, hawking,
marriages and funerals, were all announced by
cries and processions, songs and music. The lover
wore the colors of his lady; companions the

(40) emblem of their confraternity; parties and servants
the badges or blazon of their lords. Between town
and country, too, the contrast was very marked. A
medieval town did not lose itself in extensive suburbs
of factories and villas; girded by its walls, it

(45) stood forth as a compact whole, bristling with
innumerable turrets. However tall and threatening
the houses of noblemen or merchants might be, in
the aspect of the town the lofty mass of the
churches always remained dominant.

(50) The contrast between silence and sound, darkness
and light, like that between summer and winter,
was more strongly marked than it is in our
lives. The modern town hardly knows silence or
darkness in their purity, nor the effect of a solitary

(55) light or a single distant cry.

All things presenting themselves to the mind in
violent contrasts and impressive forms, lent a tone
of excitement and of passion to everyday life and
tended to produce the perpetual oscillation

(60) between despair and distracted joy, between cruelty
and pious tenderness which characterizes life
in the Middle Ages.

10. The author’s main purpose in this passage is best defined as an attempt to show how

(A) extremes of feeling and experience marked the Middle Ages

(B) the styles of the very poor and the very rich complemented each other

(C) twentieth century standards of behavior cannot be applied to the Middle Ages

(D) the Middle Ages developed out of the Dark Ages

(E) the medieval spirit languished five hundred years ago

11. According to lines 10–16, surrounding an activity with formalities makes it

(A) less important

(B) more stately

(C) less expensive

(D) more indirect

(E) less solemn

12. The author’s use of the term “formulae” (line 16) could best be interpreted to mean which of the following?

(A) set forms of words for rituals

(B) mathematical rules or principles

(C) chemical symbols

(D) nourishment for infants

(E) prescriptions for drugs

13. In line 32, “order” most nearly means

(A) command

(B) harmony

(C) sequence

(D) physical condition

(E) social class

14. According to the passage, well above the typical medieval town there towered

(A) houses of worship

(B) manufacturing establishments

(C) the mansions of the aristocracy

(D) great mercantile houses

(E) walled suburbs

15. To the author, the Middle Ages seem to be all the following EXCEPT

(A) routine and boring

(B) festive and joyful

(C) dignified and ceremonious

(D) passionate and turbulent

(E) harsh and bleak

Questions 16–24 are based on the following passage.

The following passage is excerpted from Hunger of Memory, the autobiography of Mexican-American writer Richard Rodriguez, who speaks of lessons he learned as the child of working-class immigrant parents.

I remember to start with that day in
Sacramento—a California now nearly thirty years
past—when I first entered a classroom, able to
understand some fifty stray English words.

Line (5)   The third of four children, I had been preceded
to a neighborhood Roman Catholic school by an
older brother and sister. Each afternoon they
returned, as they left in the morning, always
together, speaking in Spanish as they climbed the

(10) five steps of the porch. And their mysterious
books, wrapped in shopping-bag paper, remained
on the table next to the door, closed firmly behind
them.

An accident of geography sent me to a school

(15) where all my classmates were white, many the
children of doctors and lawyers and business
executives. All my classmates certainly must have
been uneasy on that first day of school—as most
children are uneasy—to find themselves apart

(20) from their families in the first institution of their
lives. But I was astonished.

The nun said, in a friendly but oddly impersonal
voice, “Boys and girls, this is Richard
Rodriguez.” (I heard her sound out: Rich-heard

(25) Road-ree-guess.) It was the first time I had heard
anyone name me in English. “Richard,” the nun
repeated more slowly, writing my name down in
her black leather book. Quickly I turned to see
my mother’s face dissolve in a watery blur behind

(30) the pebbled glass door.

Many years later there is something called
bilingual education—a scheme proposed in the
late 1960s by Hispanic-American social activists,
later endorsed by a congressional vote. It is a program

(35) that seeks to permit non-English-speaking
children, many from lower class homes, to use
their family language as the language of school.
(Such is the goal its supporters announce.) I hear
them and am forced to say no: It is not possible

(40) for a child—any child—ever to use his family’s
language in school. Not to understand this is to
misunderstand the public uses of schooling and to
trivialize the nature of intimate life—a family’s
“language.”

(45)      Memory teaches me what I know of these matters;
the boy reminds the adult. I was a bilingual
child, a certain kind—socially disadvantaged—
the son of working-class parents, both Mexican
immigrants.

(50) In the early years of my boyhood, my parents
coped very well in America. My father had steady
work. My mother managed at home. They were
nobody’s victims. Optimism and ambition led
them to a house (our home) many blocks from the

(55) Mexican south side of town. We lived among
gringos and only a block from the biggest,
whitest houses. It never occurred to my parents
that they couldn’t live wherever they chose. Nor
was the Sacramento of the fifties bent on teaching

(60) them a contrary lesson. My mother and father
were more annoyed than intimidated by those two
or three neighbors who tried initially to make us
unwelcome. (“Keep your brats away from my
sidewalk!”) But despite all they achieved, perhaps

(65) because they had so much to achieve, any deep
feeling of ease, the confidence of “belonging”
in public was withheld from them both. They
regarded the people at work, the faces in crowds,
as very distant from us. They were the others,

(70) los gringos. That term was interchangeable in
their speech with another, even more telling,
los americanos.

16. The family members in the passage are discussed primarily in terms of

(A) the different personalities of each

(B) the common heritage they shared

(C) the ambitions they possessed

(D) their interaction with the English-speaking world

(E) their struggle against racial discrimination

17. The author’s description of his older brother and sister’s return from school (lines 7–10) suggests that they

(A) enjoyed exploring the mysteries of American culture

(B) were afraid to speak English at home

(C) wished to imitate their English-speaking classmates

(D) readily ignored the need to practice using

English

(E) regretted their inability to make friends

18. What initially confused the author on his first day of school?

(A) His mother’s departure took him by surprise.

(B) Hearing his name in English disoriented him.

(C) His older brother and sister had told him lies about the school.

(D) He had never before seen a nun.

(E) He had never previously encountered white children.

19. In line 32, “scheme” most nearly means

(A) conspiracy

(B) diagram

(C) plan

(D) outline

(E) goal

20. The author rejects bilingual education on the grounds that

(A) allowing students to use their family’s language in school presents only trivial difficulties to teachers

(B) its champions fail to see that public education must meet public needs, not necessarily personal ones

(C) most students prefer using standard English both at home and in the classroom

(D) the proposal was made only by social activists and does not reflect the wishes of the Hispanic-American community

(E) it is an unnecessary program that puts a heavy financial burden upon the taxpayer

21. In lines 45–49, the author most likely outlines his specific background in order to

(A) emphasize how far he has come in achieving his current academic success

(B) explain the sort of obstacles faced by the children of immigrants

(C) indicate what qualifies him to speak authoritatively on the issue

(D) dispel any misunderstandings about how much he remembers of his childhood

(E) evoke the reader’s sympathy for socially disadvantaged children

22. The author’s attitude toward his parents (lines 50–72) can best be described as

(A) admiring

(B) contemptuous

(C) indifferent

(D) envious

(E) diffident

23. Which of the following statements regarding Mexican-Americans in Sacramento would be most true of the author’s experiences?

(A) They were unable to find employment.

(B) They felt estranged from the community as a whole.

(C) They found a ready welcome in white neighborhoods.

(D) They took an active part in public affairs.

(E) They were unaware of academic institutions.

24. In line 71, “telling” most nearly means

(A) outspoken

(B) interchangeable

(C) unutterable

(D) embarrassing

(E) revealing

S T O P

YOU MAY GO BACK AND REVIEW THIS SECTION IN THE REMAINING TIME, BUT DO NOT WORK IN ANY OTHER SECTION UNTIL TOLD TO DO SO.

SECTION 7

Time—25 Minutes 18 Questions

You have 25 minutes to answer the 8 multiple-choice questions and 10 student-produced response questions in this section.

For each multiple-choice question, determine which of the five choices is correct and blacken the corresponding choice on your answer sheet. You may use any blank space on the page for your work.

Notes:

• You may use a calculator whenever you think it will be helpful.

• Only real numbers are used. No question or answer on this test involves a complex or imaginary number.

• Use the diagrams provided to help you solve the problems. Unless you see the words “Note: Figure not drawn to scale” under a diagram, it has been drawn as accurately as possible. Unless it is stated that a figure is three-dimensional, you may assume it lies in a plane.

• For any function f, the domain, unless specifically restricted, is the set of all real numbers for which f (x) is also a real number.

Reference Information

123

123

  1. What is the absolute value of the product of all the integers from –6 to 3, inclusive?

(A) –120

(B) –15

(C)     0

(D)   15

(E)   120

  2. If 123 of a number is 7 more than 123 of the number, what is 123 of the number?

(A) 12

(B) 15

(C) 18

(D) 20

(E) 24

  3. If A is the set of positive multiples of 5 less than 200 and B is the set of positive multiples of 7 less than 200, how many numbers are in both set A and set B?

(A)   0

(B)   5

(C) 11

(D) 62

(E) 67

  4. If 123 + 3 = 5, then x =

(A)–9

(B) 3

(C) 7

(D) 9

(E) 26

  5. Let the lengths of the sides of a triangle be represented by x + 3, 2x – 3, and 3x – 5. If the perimeter of the triangle is 25, what is the length of the shortest side?

(A)   5

(B)   7

(C)   8

(D) 10

(E) It cannot be determined from the information given.

  6. What is the maximum number of points of intersection between a square and a circle?

(A) less than 4

(B) 4

(C) 6

(D) 8

(E) more than 8

  7. In square ABCD, vertex A is at (–1, –1) and vertex C is at (4, 2). What is the area of square ABCD?

(A)   9

(B) 15

(C) 17

(D) 25

(E) 34

  8. If f (x) = x + 5, which of the following is a solution of f (3a) + 2 = f (2a) + 3?

(A) 1

(B) 2

(C) 5

(D) 6

(E) There are no solutions.

Directions for Student-Produced Response Questions (Grid-ins)

In questions 9–18, first solve the problem, and then enter your answer on the grid provided on the answer sheet. The instructions for entering your answers are as follows:

• First, write your answer in the boxes at the top of the grid.

• Second, grid your answer in the columns below the boxes.

• Use the fraction bar in the first row or the decimal point in the second row to enter fractions and decimal answers.

123

123

• Grid only one space in each column.

• Entering the answer in the boxes is recommended as an aid in gridding, but is not required.

• The machine scoring your exam can read only what you grid, so you must grid in your answers correctly to get credit.

• If a question has more than one correct answer, grid in only one of these answers.

• The grid does not have a minus sign, so no answer can be negative.

• A mixed number must be converted to an improper fraction or a decimal before it is gridded. Enter 123 as 5/4 or 1.25; the machine will interpret 1 1/4 as 123 and mark it wrong.

• All decimals must be entered as accurately as possible. Here are the three acceptable ways of gridding

123

• Note that rounding to .273 is acceptable, because you are using the full grid, but you would receive no credit for .3 or .27, because these answers are less accurate.

  9. If 7a = (91)(13), what is the value of 123?

10. If a, b, and c are positive numbers with 123 what is the value of c when a = 44 and b = 275?

11. What is the area of a right triangle whose hypotenuse is 25 and one of whose legs is 15?

12. If the average (arithmetic mean) of a, b, c, d, and e is 95, and the average of a, b, and e is 100, what is the average of c and d?

13. If x + y = 10 and x – y = 11, what is the value of x2 – y2?

123

14. In the figure above, all of the line segments meet to form right angles. What is the perimeter of the figure?

15. If a = 2b, 3b = 4c, and 5c = 6d, what is the ratio of a to d?

16. Two circular tables have diameters of 36 inches and 24 inches, respectively. The area of the larger table is what percent more than the area of the smaller table? (Grid in your answer without a percent sign.)

17. In 1980, Elaine was 8 times as old as Adam, and Judy was 3 times as old as Adam. Elaine is 20 years older than Judy. How old was Adam in 1988?

18. Jessica created a sequence of five numbers. She chose a number for the first term and got each successive term by alternately using the following two rules: add 6 to the preceding term and double the preceding term. So the second term of Jessica’s sequence is 6 more than the first term, the third term is double the second term, the fourth term is 6 more than the third term, and the fifth term is double the fourth term. If the fifth number is 1996, what number did Jessica choose for the first term?

S T O P

YOU MAY GO BACK AND REVIEW THIS SECTION IN THE REMAINING TIME, BUT DO NOT WORK IN ANY OTHER SECTION UNTIL TOLD TO DO SO.

SECTION 8

Time—20 Minutes 19 Questions

Select the best answer to each of the following questions; then blacken the appropriate space on your answer sheet.

Each of the following sentences contains one or two blanks; each blank indicates that a word or set of words has been left out. Below the sentence are five words or phrases, lettered A through E. Select the word or set of words that best completes the sentence.

Example:

Fame is ----; today’s rising star is all too soon tomorrow’s washed-up has-been.

(A) rewarding

(B) gradual

(C) essential

(D) spontaneous

(E) transitory

123

  1. In apologizing to the uncredited photographer, the editor said that he ---- that this ---- use of copyrighted photographs had taken place.

(A) deplored..legitimate

(B) conceded..inevitable

(C) regretted..unauthorized

(D) admitted..warranted

(E) acknowledged..appropriate

  2. The herb Chinese parsley is an example of what we mean by an acquired taste: Westerners who originally ---- it eventually come to ---- its flavor in Oriental foods.

(A) relish..enjoy

(B) dislike..welcome

(C) savor..abhor

(D) ignore..detest

(E) discern..recognize

  3. Because he was ---- in the performance of his duties, his employers could not ---- his work.

(A) derelict..quarrel over

(B) dilatory..grumble at

(C) undisciplined..object to

(D) assiduous..complain about

(E) mandatory..count on

  4. British ---- contemporary art has been an obstacle even for modern artists now revered as great, such as Francis Bacon and Lucian Freud, who were ---- for years before winning acceptance.

(A) veneration of..eulogized

(B) indifference to..dismissed

(C) disdain for..lauded

(D) ignorance of..studied

(E) intolerance of..vindicated

  5. The biochemistry instructor urged that we take particular care of the ---- chemicals to prevent their evaporation.

(A) insoluble

(B) superficial

(C) extraneous

(D) volatile

(E) insipid

  6. It is said that the custom of shaking hands originated when primitive men held out empty hands to indicate that they had no ---- weapons and were thus ---- disposed.

(A) lethal..clearly

(B) concealed..amicably

(C) hidden..harmfully

(D) murderous..ill

(E) secret..finally

The questions that follow the next two passages relate to the content of both, and to their relationship. The correct response may be stated outright in the passage or merely suggested.

Questions 7–19 are based on the following passages.

The following passages are adapted from essays on detective fiction, often known as mysteries. In the first, the poet W. H. Auden discusses the detective story’s magic formula. In the second, historian Robin Winks assesses what we do when we read mysteries.

Passage 1

The most curious fact about the detective story
is that it makes its greatest appeal precisely to
those classes of people who are most immune to
other forms of daydream literature. The typical

Line (5) detective story addict is a doctor or clergyman or
scientist or artist, i.e., a fairly successful professional
man with intellectual interests and well-read
in his own field, who could never stomach
the Saturday Evening Post or True Confessions or

(10) movie magazines or comics.

It is sometimes said that detective stories are
read by respectable law-abiding citizens in order
to gratify in fantasy the violent or murderous
wishes they dare not, or are ashamed to, translate

(15) into action. This may be true for readers of
thrillers (which I rarely enjoy), but it is quite false
for the reader of detective stories. On the contrary,
the magical satisfaction the latter provide
(which makes them escape literature, not works

(20) of art) is the illusion of being dissociated from the
murderer.

The magic formula is an innocence which is
discovered to contain guilt; then a suspicion of
being the guilty one; and finally a real innocence

(25) from which the guilty other has been expelled, a
cure effected, not by me or my neighbors, but by
the miraculous intervention of a genius from outside
who removes guilt by giving knowledge of guilt.
(The detective story subscribes, in fact, to

(30) the Socratic daydream: “Sin is ignorance.”)

If one thinks of a work of art which deals with
murder, Crime and Punishment for example, its
effect on the reader is to compel an identification
with the murderer which he would prefer not to

(35) recognize. The identification of fantasy is always
an attempt to avoid one’s own suffering: the
identification of art is a compelled sharing in the suffering
of another. Kafka’s The Trial is another
instructive example of the difference between a

(40) work of art and the detective story. In the latter it
is certain that a crime has been committed and,
temporarily, uncertain to whom guilt should be
attached; as soon as this is known, the innocence
of everyone else is certain. (Should it turn out that

(45) after all no crime has been committed, then all
would be innocent.) In The Trial, on the other
hand, it is the guilt that is certain and the crime
that is uncertain; the aim of the hero’s investigation
is not to prove his innocence (which would

(50) be impossible for he knows he is guilty), but to
discover what, if anything, he has done to make
himself guilty. K, the hero, is, in fact, a portrait of
the kind of person who reads detective stories for
escape.

(55)      The fantasy, then, which the detective story
addict indulges is the fantasy of being restored to
the Garden of Eden, to a state of innocence,
where he may know love as love and not as the
law. The driving force behind this daydream is

(60) the feeling of guilt, the cause of which is
unknown to the dreamer. The fantasy of escape is
the same, whether one explains the guilt in
Christian, Freudian, or any other terms. One’s
way of trying to face the reality, on the other

(65) hand, will, of course, depend very much on one’s
creed.

Passage 2

Detective fiction creates for us an anonymity;
within it, we may constitute the last law on earth,
making decisions (to be “proved” right or wrong)

(70) as we go, responsible for them, tricked, disappointed,
triumphant, joyful, honest as to our mistakes,
setting the record straight. As we make
leaps of faith between evidence and decision in
our daily lives—to board this bus, to choose that

(75) doctor, to add these pounds—so we make leaps of
faith between evidence and conclusion, through
the public historiography and the private autobiography
that we read. We learn how to define
evidence, to use up our intellectual shoe leather in

(80) pursuit of an operable truth, to take joy from the
receding horizon and pleasure in the discovery
that the answer has not yet been found, that there
is more work to be done. We learn that what
people believe to be true is as important as the objective

(85) truth defined by the researcher/detective. In
Marlowe and Archer we meet people who have
no use for their conclusions, no desire for
vengeance, who know that society will supply the
uses while they may engage in the happy ambiguity

(90) of simply finding the facts, which, inert, take
on life when embedded in a context of cause and
effect.

Ultimately one reads detective fiction because
it involves judgments—judgments made, passed

(95) upon, tested. In raising questions about purpose, it
raises questions about cause and effect. In the
end, like history, such fiction appears to, and
occasionally does, decode the environment;
appears to and occasionally does tell one what to

(100) do; appears to and occasionally does set the
record straight. Setting the record straight ought
to matter. Detective fiction, in its high seriousness,
is a bit like a religion, in pursuit of truths
best left examined at a distance. As with all fine

(105) literature, history, philosophy, as with the written
word wherever employed creatively, it can lead
us to laughter in our frustration, to joy in our
experience, and to tolerance for our complexities.
It begins as Hawthorne so often does, and as the

(110) best of historians do, with a personal word,
diffident, apparently modest, in search of the subject
by asking, What is the question? It ends, as historians
who have completed their journey often do,
with an authoritative tone, the complex explained,

(115) the mystery revealed.

  7. In line 1, “curious” most nearly means

(A) inquisitive

(B) unusual

(C) sensitive

(D) prying

(E) salutary

  8. The opening paragraph of Passage 1 suggests that the author would consider True Confessions and movie magazines to be

(A) sources of factual data about society

(B) worthwhile contemporary periodicals

(C) standard forms of escapist literature

(D) the typical literary fare of professionals

(E) less addictive than detective fiction

  9. The author of Passage 1 asserts that readers of detective fiction can most accurately be described as

(A) believers in the creed of art for art’s sake

(B) people bent on satisfying an unconscious thirst for blood

(C) dreamers unable to face the monotony of everyday reality

(D) persons seeking momentary release from a vague sense of guilt

(E) idealists drawn to the comforts of organized religion

10. In line 14, “translate” most nearly means

(A) decipher

(B) move

(C) explain

(D) convey

(E) convert

11. Which best describes what the author is doing in citing the example of Kafka’s The Trial (lines 46–54)?

(A) Dramatizing the plot of a typical detective story

(B) Analyzing its distinctive qualities as a work of art

(C) Refuting a common opinion about readers of detective fiction

(D) Demonstrating the genius of the outside investigator

(E) Discrediting a theory about Kafka’s narrative

12. In Passage 1, the author’s attitude toward detective fiction can best be described as one of

(A) fastidious distaste

(B) open skepticism

(C) profound veneration

(D) aloof indifference

(E) genuine appreciation

13. In context, “use up our intellectual shoe leather” (line 79) suggests that readers of mysteries

(A) suffer in the course of arriving at the truth

(B) are attempting to escape from overly strenuous intellectual pursuits

(C) work hard mentally, much as detectives do physically

(D) have only a limited supply of time to devote to detective fiction

(E) grow hardened to crime in the course of their reading

14. In lines 78–83, the author of Passage 2 finds the prospect of additional work

(A) burdensome

(B) unexpected

(C) unfounded

(D) delightful

(E) deceptive

15. Passage 2 suggests that Marlowe and Archer are most likely

(A) murder victims

(B) fictional detectives

(C) prominent novelists

(D) literary scholars

(E) rival theorists

16. In line 106, “employed” most nearly means

(A) hired

(B) used

(C) commissioned

(D) remunerated

(E) labored

17. According to lines 109–112, the detective story starts by

(A) setting the record straight

(B) simplifying the difficulties of the case

(C) humanizing the investigating detective

(D) introducing the characters under suspicion

(E) defining the problem to be solved

18. Both passages are primarily concerned with the question of

(A) whether detective stories gratify a taste for violence

(B) why people enjoy reading detective fiction

(C) how detectives arrive at their conclusions

(D) why some people resist the appeal of escapist literature

(E) whether detective stories can be considered works of art

19. The author of Passage 1 would most likely react to the characterization of detective fiction presented in lines 93–115 by pointing out that

(A) reading detective fiction is an escape, not a highly serious pursuit

(B) other analyses have shown the deficiencies of this characterization

(C) this characterization reflects the author’s lack of taste

(D) this characterization is neither original nor objective

(E) the realities of the publishing trade justify this characterization

S T O P

YOU MAY GO BACK AND REVIEW THIS SECTION IN THE REMAINING TIME, BUT DO NOT WORK IN ANY OTHER SECTION UNTIL TOLD TO DO SO.

SECTION 9

Time—20 Minutes 16 Questions

For each problem in this section determine which of the five choices is correct and blacken the corresponding choice on your answer sheet. You may use any blank space on the page for your work.

Notes:

• You may use a calculator whenever you think it will be helpful.

• Only real numbers are used. No question or answer on this test involves a complex or imaginary number.

• Use the diagrams provided to help you solve the problems. Unless you see the words “Note: Figure not drawn to scale” under a diagram, it has been drawn as accurately as possible. Unless it is stated that a figure is three-dimensional, you may assume it lies in a plane.

• For any function f, the domain, unless specifically restricted, is the set of all real numbers for which f (x) is also a real number.

Reference Information

123

123

  1. If 2x – 1 = 9, what is 10x – 5?

(A) 35

(B) 45

(C) 55

(D) 75

(E) 95

123

Note: Figure not drawn to scale

  2. If in the figure above, 1231 || 1232, which of the following statements about a + b is true?

(A) a + b < 180

(B) a + b = 180

(C) 180 < a + b ≤ 270

(D) 270 < a + b ≤ 360

(E) It cannot be determined from the information given.

  3. Which of the following expressions has the greatest value?

(A) 4 × 4 ÷ 4 + 4

(B) 4 ÷ 4 × 4 + 4

(C) 4 × 4 – 4 × 4

(D) 4 ÷ 4 + 4 × 4

(E) 4 + 4 × 4 – 4

  4. Hoover High School has 840 students, and the ratio of the number of students taking Spanish to the number not taking Spanish is 4:3. How many of the students take Spanish?

(A) 280

(B) 360

(C) 480

(D) 560

(E) 630

  5. Which of the following CANNOT be expressed as the sum of three consecutive integers?

(A) 18

(B) 24

(C) 28

(D) 33

(E) 36

  6. If 123 then n =

(A)   2

(B)   4

(C)   8

(D)   9

(E) 19.25

123

  7. In the figure above, if k || 123, what is the value of y?

(A) 40

(B) 45

(C) 50

(D) 60

(E) 65

  8. Consider the sequence 1, 2, 3, 1, 2, 3, 1, 2, 3, … . What is the sum of the first 100 terms?

(A) 100

(B) 180

(C) 198

(D) 199

(E) 200

  9. If, for any numbers a and b123 b represents the average (arithmetic mean) of a and b, which of the following MUST be true?

  I. 123

 II. 123

III. 123

(A) I only

(B) II only

(C) I and II only

(D) II and III only

(E) I, II, and III

123

10. If the selling price of model B is 60% more than the selling price of model A, what percent of the total sales do the sales of model A represent?

(A) 25%

(B) 36%

(C) 40%

(D) 50%

(E) 60%

123

11. In the figure above, ABCDBEFG, and DHIJ are squares with AB = 2(DH) and DH = 2(BE). If a point is chosen at random inside square ABCD, what is the probability it will be in the shaded region?

(A) 123

(B) 123

(C) 123

(D) 123

(E) 123

12. If x = 123 (x + y), which of the following is an expression for x in terms of y ?

(A) 123y

(B) y

(C) 123y

(D) 2y

(E) 3y

123

13. In the figure above, A and B are points on circle O and PA and PB are tangent to the circle. If m 123P = 50, what is the degree measure of 123AOB (not shown)?

(A)   40

(B)   50

(C)   90

(D) 130

(E) 140

123

14. Which of the following is the equation of the line shown in the figure above?

(A) y = – 123 x + 2

(B) y = 123x + 2

(C) y = – 123 x + 4

(D) y = 2x + 4

(E) y = –2x + 4

15. Let P and Q be points 2 inches apart, and let A be the area, in square inches, of a circle that passes through P and Q. Which of the following is the set of all possible values of A?

(A) 0 < A

(B) 0 < A 123 123

(C) A = 123

(D) A > 123

(E) 123 123

16. If x + 2y = a and x – 2y = b, which of the following is an expression for xy?

(A) ab

(B) 123

(C) 123

(D) 123

(E) 123

S T O P

YOU MAY GO BACK AND REVIEW THIS SECTION IN THE REMAINING TIME, BUT DO NOT WORK IN ANY OTHER SECTION UNTIL TOLD TO DO SO.

SECTION 10

Time—10 Minutes 14 Questions

For each of the following questions, select the best answer from the choices provided and fill in the appropriate circle on the answer sheet.

Some or all parts of the following sentences are underlined. The first answer choice, (A), simply repeats the underlined part of the sentence. The other four choices present four alternative ways to phrase the underlined part. Select the answer that produces the most effective sentence, one that is clear and exact, and blacken the appropriate space on your answer sheet. In selecting your choice, be sure that it is standard written English, and that it expresses the meaning of the original sentence.

Example:

The first biography of author Eudora Welty came out in 1998 and she was 89 years old at the time.

(A) and she was 89 years old at the time

(B) at the time when she was 89

(C) upon becoming an 89 year old

(D) when she was 89

(E) at the age of 89 years old

123

  1. In the four chapels of Santa Croce, Giotto painted frescoes and they portrayed the lives of the saints.

(A) frescoes and they portrayed

(B) frescoes, being portrayals of

(C) frescoes, they portrayed

(D) frescoes that portrayed

(E) frescoes because they portrayed

  2. The debate coach, together with the members of the winning team, is traveling to Washington for the awards ceremony.

(A) together with the members of the winning team, is traveling

(B) along with the members of the winning team, they are traveling

(C) along with the members of the winning team, are traveling

(D) together with the members of the winning team, are traveling

(E) together with the members of the winning team, are to travel

  3. By establishing strict rules of hygiene in maternity wards, Ignaz Semmelweis saved many women from dying of childbed fever, this was a fate that many expectant mothers feared.

(A) fever, this was a fate that many expectant mothers feared

(B) fever, since many expectant mothers feared this was their fate

(C) fever, it was a fate of which many expectant mothers were afraid

(D) fever, because many expectant mothers feared this fate

(E) fever, a fate that many expectant mothers feared

  4. Veterans of World War II received greater support from the public than the Korean and Vietnam Wars.

(A) than

(B) than did

(C) than did veterans of

(D) than from the support of

(E) than from the

  5. Nowadays airport security guards have the right to search people’s bags who act in a suspicious manner.

(A) people’s bags who act

(B) persons’ bags who act

(C) the bags of people who act

(D) the bags of persons that act

(E) personal bags which act

  6. The clipper ship was the fastest ocean-going vessel of its time; it ruled the waves only briefly, however, before the faster and more reliable steamship took its place.

(A) time; it ruled the waves only briefly, however,

(B) time, for it ruled the waves only briefly

(C) time; however, ruling the waves only briefly

(D) time, having ruled the waves only briefly, however,

(E) time, but was ruling the waves only briefly, however,

  7. The real estate reporter maintained that housing prices in San Francisco were higher than any other city in the country.

(A) higher than any other city

(B) higher than every other city

(C) the highest of those of any other city

(D) higher than those in any other city

(E) higher than any city

  8. During the eighteenth century, inoculations against smallpox became increasingly popular among the English upper classes although to the lower classes it remained mysterious and therefore threatening.

(A) although to the lower classes it

(B) because to the lower classes it

(C) although to the lower classes such inoculations

(D) however, to the lower classes the inoculations

(E) although among the lower classes it

  9. With the rift between the two sides apparently widening, analysts said that they considered the likelihood of a merger between the two corporations to be negligible.

(A) considered the likelihood of a merger between the two corporations to be negligible

(B) considered it was likely a merger between the two corporations being negligible

(C) considered the two corporations’ merger likely to be negligible

(D) considered the likelihood of the two corporations merging between them to have been negligible

(E) considered between the two corporations such a merger to be negligible

10. Gold was discovered at Sutter’s Mill in 1848, and the prospectors who flocked to the gold fields are known not as the forty-eighters but as the forty-niners.

(A) Gold was discovered at Sutter’s Mill in 1848, and

(B) They discovered gold at Sutter’s Mill in 1848, and

(C) Although gold was discovered at Sutter’s Mill in 1848,

(D) Upon the discovery of gold at Sutter’s Mill in 1848,

(E) Because gold was discovered at Sutter’s Mill in 1848,

11. Once a leading light of the Harlem Renaissance, the revived interest in African-American literary pioneers rescued Zora Neale Hurston from decades of obscurity.

(A) the revived interest in African-American literary pioneers rescued Zora Neale Hurston from decades of obscurity

(B) through the revived interest in African-American literary pioneers, Zora Neale Hurston was rescued from decades of obscurity

(C) Zora Neale Hurston’s rescue from decades of literary obscurity was due to the revived interest in African-American literary pioneers

(D) Zora Neale Hurston was rescued from decades of literary obscurity by the revived interest in African-American literary pioneers

(E) Zora Neale Hurston was rescued from decades of literary obscurity by reviving the interest in African-American literary pioneers

12. The historians of geography and cartography seem more interested in their maps than in the explorers who went into the field, often at great risk, to get the information that these maps contain.

(A) explorers who went into the field, often at great risk, to get the information that these maps contain

(B) explorers that went into the field, often at great risk, to get the information these maps containing

(C) explorers going into the field, often greatly risking, and they got the information that these maps contain

(D) explorers who went into the field to get the information that these maps often contain at great risk

(E) explorers often at great risk that were the ones who went into the field to get the information contained in these maps

13. Employment statistics indicate that the percentage of workers who found jobs in the fall quarter is lower than the spring.

(A) workers who found jobs in the fall quarter is lower than the spring

(B) workers that found jobs in the fall quarter is lower than the percentage in the spring

(C) workers who found jobs in the fall quarter is lower than the equivalent percentage in the spring

(D) workers who had found jobs in the fall quarter is lower than the spring

(E) workers finding jobs in the fall quarter is lower than the spring quarter

14. Most of the free libraries founded by Andrew Carnegie were located in communities where there were hardly no other cultural institutions available to members of the working classes.

(A) Most of the free libraries founded by Andrew Carnegie were located in communities where there were hardly no other cultural institutions

(B) Of the free libraries founded by Andrew Carnegie, most were located in communities in which there were hardly no other cultural institutions

(C) Most free libraries that were founded by Andrew Carnegie he located in communities where hardly any other cultural institutions were

(D) Andrew Carnegie founded mostly free libraries located in communities where there were hardly any other cultural institutions

(E) Most of the free libraries founded by Andrew Carnegie were located in communities where there were hardly any other cultural institutions

S T O P

YOU MAY GO BACK AND REVIEW THIS SECTION IN THE REMAINING TIME, BUT DO NOT WORK IN ANY OTHER SECTION UNTIL TOLD TO DO SO.

Answer Key

Note: The letters in brackets following the Mathematical Reasoning answers refer to the sections of Chapter 9 in which you can find the information you need to answer the questions. For example, 1. C [E] means that the answer to question 1 is C, and that the solution requires information found in Section 9-E: Averages.

Section 2      Critical Reading

  1. D

  2. E

  3. C

  4. D

  5. D

  6. E

  7. B

  8. D

  9. C

10. E

11. C

12. E

13. C

14. A

15. C

16. D

17. E

18. B

19. B

20. A

21. E

22. A

23. C

24. D

Section 3      Mathematical Reasoning

  1. C [G]

  2. D [I]

  3. E [C]

  4. E [A]

  5. B [I]

  6. B [I]

  7. C [O]

  8. B [A]

  9. C [C]

10. D [A]

11. C [K]

12. E [A]

13. E [D, P]

14. A [K]

15. B [J, K]

16. A [D]

17. B [M]

18. D [P]

19. A [H]

20. C [E]

Section 4      Writing Skills

  1. D

  2. E

  3. B

  4. E

  5. C

  6. B

  7. A

  8. E

  9. C

10. E

11. A

12. B

13. A

14. B

15. B

16. C

17. C

18. C

19. B

20. B

21. C

22. D

23. E

24. E

25. C

26. D

27. A

28. C

29. E

30. C

31. B

32. E

33. A

34. B

35. D

Section 5

On this test, Section 5 was the experimental section. It could have been an extra critical reading, mathematics, or writing skills section. Remember: on the SAT you take, the experimental section may be any section from 2 to 7.

Section 6 Critical Reading

  1. E

  2. C

  3. C

  4. D

  5. E

  6. D

  7. C

  8. B

  9. C

10. A

11. B

12. A

13. E

14. A

15. A

16. D

17. D

18. B

19. C

20. B

21. C

22. A

23. B

24. E

Section 7 Mathematical Reasoning

Multiple-Choice Questions

1. C [A]

2. D [G]

3. B [A]

4. C [G]

5. B [K, G]

6. D [L]

7. C [K, N]

8. A [R]

Grid-in Questions

123

Section 8 Critical Reading

  1. C

  2. B

  3. D

  4. B

  5. D

  6. B

  7. B

  8. C

  9. D

10. E

11. B

12. E

13. C

14. D

15. B

16. B

17. E

18. B

19. A

Section 9 Mathematical Reasoning

  1. B [G]

  2. E [I]

  3. D [A]

  4. C [D]

  5. C [A]

  6. B [A]

  7. E [I, G]

  8. D [P]

  9. C [E]

10. B [Q, C]

11. D [K, O]

12. D [G]

13. D [L]

14. A [N]

15. E [L]

16. E [G]

Section 10 Writing Skills

  1. D

  2. A

  3. E

  4. C

  5. C

  6. A

  7. D

  8. C

  9. A

10. C

11. D

12. A

13. C

14. E

Score Your Own SAT Essay

Use this table as you rate your performance on the essay-writing section of this Model Test. Circle the phrase that most accurately describes your work. Enter the numbers in the scoring chart below. Add the numbers together and divide by 6 to determine your total score. The higher your total score, the better you are likely to do on the essay section of the SAT.

Note that on the actual SAT two readers will rate your essay; your essay score will be the sum of their two ratings and could range from 12 (highest) to 2 (lowest). Also, they will grade your essay holistically, rating it on the basis of their overall impression of its effectiveness. They will not analyze it piece by piece, giving separate grades for grammar, vocabulary level, and so on. Therefore, you cannot expect the score you give yourself on this Model Test to predict your eventual score on the SAT with any great degree of accuracy. Use this scoring guide instead to help you assess your writing strengths and weaknesses, so that you can decide which areas to focus on as you prepare for the SAT.

Like most people, you may find it difficult to rate your own writing objectively. Ask a teacher or fellow student to score your essay as well. With his or her help you should gain added insights into writing your 25-minute essay.

123

Self-Scoring Chart

Scoring Chart (Second Reader)

For each of the following categories,
rate the essay from 1 (lowest)
to 6 (highest)

For each of the following categories,
rate the essay from 1 (lowest)
to 6 (highest)

Position on the Topic

123

Position on the Topic

123

Organization of Evidence

123

Organization of Evidence

123

Sentence Structure

123

Sentence Structure

123

Level of Vocabulary

123

Level of Vocabulary

123

Grammar and Usage

123

Grammar and Usage

123

Overall Effect

123

Overall Effect

123

TOTAL

123

TOTAL

123

(To get a score, divide the total by 6)

123

(To get a score, divide the total by 6)

123

Calculate Your Raw Score

Critical Reading

123

Mathematical Reasoning

123

Writing Skills

123

Evaluate Your Performance

123

Identify Your Weaknesses

123

Identify Your Weaknesses

123

Identify Your Weaknesses

123

ANSWERS EXPLAINED

Section 2 Critical Reading

  1. D.    Mitchell is optimistic about the future of African-Americans in ballet. However, he is not blindly optimistic. Instead, he is realistic about the problems blacks face.

Remember to watch for signal words that link one part of the sentence with another. The use of “Although” in the opening clause sets up a contrast. The missing word must be an antonym for “optimistic.”

(Contrast Signal)

  2. E.    Frustrations or limitations are by definition not satisfying.

Again, remember to watch for signal words that link one part of the sentence with another. The use of “Despite” in the opening clause sets up a contrast. The missing word must be an antonym for “stirring and satisfying.” Note, too, that you are looking for a word with negative associations.

(Contrast Signal)

  3. C.    Derivative means unoriginal. Unoriginal work derives from or comes from the work of others. The moviemaker is unoriginal despite his reputation for skill or expertise.

The word “sadly” is your clue to look for a negative word to fill in the second blank. Therefore, you can eliminate any word with positive associations.

(Definition)

  4. D.    One crowns a career with a triumph, in this case an unsurpassable performance.

Remember, before you look at the choices, to read the sentence and think of a word that makes sense; magnificent, superlative, and matchless come to mind. Note that you are looking for a word with positive associations. Therefore, you can eliminate any word with negative ones. Choices A, B, C, and E all have negative associations. Only Choice D can be correct.

(Examples)

  5. D.    Students of genetics will have to turn or resort to respected journals where relevant information is found.

Choice A is incorrect. Excellent journals would be unlikely to ignore relevant materials.

Choice B is incorrect. An interested student of genetics would enjoy reading an excellent journal in the field. Such a student would not be likely to suffer through it.

Choice C is incorrect. Pertinent material logically would be represented in an excellent technical journal.

Choice E is incorrect. We would be unlikely to complain about excellent technical journals.

(Argument Pattern)

  6. E.    Rather than alleviating or easing problems, rent control may worsen or exacerbate them.

The signal words “rather than” indicate that the missing word must be an antonym or nearantonym for “alleviate.”

You can immediately eliminate resolvediminish and minimize, which make no sense in the context.

(Contrast Signal)

  7. B.    This sentence sets up a contrast between Dickens’s image of country dancing and Sharp’s. Dickens views country dances as “lively, even (something).” In this context, even acts as an intensifier: it points up just how verylively these country dances could get. The first missing word must be a synonym for very lively.

Look at the first word of each answer choice. Sophisticatedprudish, and lewd are highly unlikely choices as synonyms for very lively.

You can immediately eliminate Choices A, C, and D. Enjoyable also seems an unlikely choice: liveliness, carried to an extreme, does not by definition become more enjoyable. You probably can eliminate Choice E. Only Choice B is left. Could lively dances grow so lively that they might become disorderly, rowdy affairs? They could.

Consider the second word in Choice B. Sharp’s image of dancing is excessively formal. It is not rowdy. It suggests more decorum and propriety than the behavior Dickens described. The correct answer is Choice B.

(Contrast Pattern)

  8. D.    Given treachery on the part of their allies, it is likely that the natives would seek vengeance or retribution. It is also likely that they would feel justified in doing so.

Test the first word in each answer choice.

Betrayed natives who have become bitter would be unlikely to seek acquiescence (agreement), magnanimity (generosity of spirit; nobility of mind), or exoneration        (vindication). You can immediately rule out Choices A, B, and E.

(Cause and Effect Pattern)

  9. C.    The outdoor playgrounds where the young athletes work on their game are not sheltered (they play in the heat of the afternoon sun and in the bitter cold of winter). You can eliminate Choice B. With their netless rims and lack of illumination (not to mention proper flooring), these playgrounds clearly are neither professional nor well designed. You can eliminate Choices A and D. Rather than being seldom accessible, the playgrounds are all too accessible: the young players haunt them by day and night. You can eliminate Choice E. Only Choice C is left. It is the correct answer. As basketball facilities go, these playgrounds are clearly rudimentary (crude; undeveloped).

10. E.    The salvation the young players hope for would come “in the form of a contract with the NBA,” that is, the National Basketball Association. Young people often look to professional sports as a way out of poverty. The salvation these basketball players dream of is their financial deliverance.

11. C.    With comic lines such as “even the fleas would desert him for a velocipede,” this portrait of the not-so-wily coyote clearly illustrates humorous exaggeration.

12. E.    In typically exaggerated fashion, Twain claims that the meanest, most contemptible creature would look down on the lowly coyote.

13. C.    Consider the activities Dillard describes Darwin participating in. He threw marine iguanas into the ocean, just for the sport of it; he went for rides on the backs of giant tortoises, as children today do on visits to the zoo. Thus, Dillard’s initial picture of Darwin gives us a sense of his youthful playfulness.

14. A.    In 1844, Darwin has come to the conclusion that species are not immutable (unable to change). He now believes that species do change. This conclusion is the opposite of what he originally thought. In other words, he originally viewed species as unchanging.

In answering questions about passages containing parenthetical comments, you may find it useful to read the passage without the remarks in parentheses. (“I am almost convinced . . . that species are not . . . immutable.”) Once you have the basic idea, go back to the parenthetical comments to see how they modify what is being stated.

Note, by the way, the importance of negative words and prefixes here. Darwin says he almost believes that species are not immutable.

In other words, he almost believes that species are mutable, able to evolve or change.

15. C.    The observations that led Darwin to formulate the theory of evolution—the records Darwin made of the minute differences among the different types of finch—took place in the Galapagos. Thus, Dillard asserts that Darwin’s theory of evolution and all it has meant to modern society began right there.

16. D.    A single species of finch had been taken and modified to meet different purposes, to serve different ends. Again, treat this vocabulary-incontext question as if it were a sentence completion exercise. Go back to the original sentence and substitute each of the different answer choices for the word in quotes.

17. E.    To say psssssh psssssh psssssh psssssh until you run out of breath is an everyday sort of thing: if you kept on breathing out that way, you’d naturally exhaust your supply of oxygen. However, to say it until an island runs out of birds—that’s something else again. The idea that an island could run out of birds, that bird after bird after bird could come swooping down from the sky until there were no more birds left anywhere around—this image emphasizes the overwhelming response of the birds.

18. B.    “Lighting” here means landing or coming to rest on a branch.

19. B.    As each bird lands or touches down on a branch, the impact rocks the slender tree. It is the rhythm made by these repeated impacts that Dillard feels as “a rough pulse from the tree’s thin trunk” against her hand.

20. A.    Dillard spends one paragraph describing the finches as drab, dull creatures. In the next paragraph she shows how they can be differentiated by their distinctive beaks. Thus, her description of the finches chiefly serves to contrast their overall drabness with their variety in one specific aspect.

21. E.    Short beaks are described as good for nabbing or catching insects. Long beaks are described as good for probing or poking deep into plants. Chisel beaks are described as good for digging grubs out of trees. What do these beaks have in common? They all enable the birds to reach nourishment; they have evolved to meet a particular need.

22. A.    Since the Galapagos are relatively close to the mainland, some mainland birds might have ventured or taken the risk of flying there.

23. C.    Strip down the sentence, rephrasing it in its shortest form: “You see . . . on the finches . . . a fluorescent spray of horn.” Horn here means a hard projection, in this case a bright, shiny one. It is the author’s poetic way of referring to the birds’ shiny beaks.

24. D.    Use the process of elimination to find the correct answer to this question.

In the parenthetical comment beginning “In other words” (lines 88–90), the author restates an assertion. Therefore, you can eliminate Choice A.

The author compares an archipelago to an arpeggio. Therefore, you can eliminate Choice B.

The author defines an arpeggio as “a rapid series of distinct but related notes.” Therefore, you can eliminate Choice C.

In the parenthetical comment beginning “In other words” (lines 88–90), the author describes a sequence of events. Therefore, you can eliminate Choice E.

Only Choice D is left. It is the correct answer.

The author never refutes or disproves an argument.

Section 3 Mathematical Reasoning

In each mathematics section, for many problems, an alternative solution, indicated by two asterisks (**), follows the first solution. When this occurs, one of the solutions is the direct mathematical one and the other is based on one of the tactics discussed in Chapter 8 or 9.

  1. C.    Use the six-step method of TACTIC G1 on the given equation, 5c + 3 = 3c + 5:

5c + 3 = 3c + 5 123 2c + 3 = 5.

So, 2c = 2 and c = 1.

**Use TACTIC 5. Backsolve, starting with C.

  2. D.    Since 123ABC is an isosceles right triangle, x = 45; also, y = 90, since it is a corner of square CDEF. Therefore, a + b = 360 – (45 + 90) = 360 – 135 = 225.

**Use TACTIC 2: trust the diagram. Clearly, 135 and 180 are too small, and 270 is too large. Guess between 210 and 225.

123

  3. E. 123    Since 74% + 18% = 92%, the $216 spent on the party represents the other 8% of the money raised. Then 0.08m = 216 123 m = 216 ÷ 0.08 = 2700.

  4. E.    Here, 123

**Use TACTIC 7. Pick an easy-to-use number, say 2. (Note that 1 would not be a good choice because then each of the five choices would be 1.) Then, 123 Only Choice E equals 1 when n = 2.

  5. B.    Since vertical angles have the same measure (KEY FACT I4), c = dd = a, and b = a – b 123 a = 2b. Therefore, c = d = a = 2b. Also, the sum of the measures of all six angles is 360° (KEY FACT I3), so a + b + c + d + a – bd = 2a + c + 2d = 360. Replacing cd, and a by 2b yields 10b = 360 123 b = 36.

  6. B.    Since the two angles, x and y, form a straight angle, x + y = 180 (KEY FACT I2). Also, it is given that x = y + 150. Therefore, (y + 150) + y = 180 123 2y + 150 = 180, and so 2y = 30 123 y = 15.

**Use TACTIC 5: backsolve. Start with 20, Choice C. If y = 20, then x = 170, but 20 + 170 = 190, which is too large. Eliminate C, D, and E, and try A and B. B works.

  7. C.    There is a total of 1 + 2 + 3 + 4 + 5 + 6 = 21 slips of paper. Since odd numbers are written on 1 + 3 + 5 = 9 of them, the probability of drawing an odd number is 123

  8. B.    The given equation can be written as a3 = 3a. Since a is positive, we can divide each side by aa2 = 3. There is only 1 positive number that satisfies this equation: 123. (Note that 0 and – 123 also satisfy the original equation, but neither of these is positive.)

  9. C. 123    Jose made a 25% profit, so if he bought the painting for x, he sold it for x + 0.25x = 1.25x = 2000.

So, x = 2000 ÷ 1.25 = 1600.

**If you get stuck or don’t know how to solve this problem, eliminate the absurd answer choices and guess. Selling the painting for $2000, Jose made a profit, so he bought it for less. Eliminate choices D and E and guess.

10. D. 123    Since = 123 the fraction reduces to 10 × 9 = 90. Now, evaluate the three choices.

I: 5! – 4! – 3! = 120 – 24 – 6 = 90 (true).

II: 123 = 5 (false).

III: 15(3!) = 15(6) = 90 (true).

I and III only are true.

11. C.    Use TACTIC 1: draw a diagram and label it. Use the Pythagorean theorem to find d, the length of the diagonal: a2 + (2a)2 = d2 123 a2 + 4a2 = d2 123 5a2 = d2 123 d = a 123.

123

**Use TACTIC 6. Let a = 1. Then use TACTIC 1, and draw the rectangle to scale; that is, let the width be 1 and the length be 2. Now use TACTIC 2: trust your eyes. Clearly, the diagonal is longer than 2 and shorter than 3 (the width plus the length is 3). The answer must be a 123, the only choice between 2 and 3, when a = 1.

123

12. E.    • Could exactly one of ab, and c be an integer? 123

• Could exactly two of ab, and c be integers? 123

• Could all three be integers? Yes again: (1)(1)(1) = 1. Be careful: the question does not require ab, and c to be different numbers.

Statements IIIand III are all true.

13. E.    Draw a Venn diagram. Of the 100 students taking chemistry, 20 take biology, and 80 don’t; they take only chemistry. Similarly, of the 80 students taking biology, 20 also take chemistry, and 60 take only biology. The desired ratio is 80:60 = 4:3 = 123

123

14. A.    Let S be a side of the large square, and s a side of the small square. The area of the white region is just s2, whereas the area of the shaded region is S2 – s2. Therefore, 123 The ratio of the side of the large square to the side of the small square is 123.

**Redraw the diagram with the small square in the corner of the large one. If the ratio were 2:1, the white area would be much smaller than the shaded area, so the small square must be larger and the ratio is less than 2:1. Only 123 is less than 2:1.

123

15. B. 123    AC is the hypotenuse of a 30-60-90 right triangle. By KEY FACT J11, the length of CD, the leg opposite the 30° angle, is 5 (half the hypotenuse), and AD = 5 123 . Then the area of the rectangle is 5 × 5 123 = 25 123 .

**Use TACTIC 2: trust the diagram. Side AD is surely shorter than the hypotenuse, say 8. Side CD is about half the hypotenuse, say 5. Then the area is about 40. Eliminate C, D, and E. Use your calculator to evaluate A ( 12335.4) and B ( 12343.3) and then guess.

16. A.    Set up a proportion: 123

123 123

Then, 123

17. B.    The formulas for the volumes of a rectangular solid and a cylinder are V123wh and V = 123r2h, respectively. (Remember that these formulas are given to you on the first page of every SAT math section.) The volume of container I is (4)(4)(5) = 80 cubic inches. Since the diameter of container II is 4, its radius is 2, and so its volume is 123(22)(5) = 20123. The difference in volumes is 80 – 20123 = 20(4 – π).

**If you don’t know, or can’t use, the formulas, you must guess. If you knew the formulas, you could answer the question, so eliminate E. Also, D is out since 1 – 123 is negative. If that’s all you know, guess among A, B, and C. If you know that the volume of the rectangular solid is 80, but you don’t know the volume of the cylinder, eliminate C, which is over 160, and guess between A and B. Don’t leave this question out.

18. D.    Since the number of cells doubles every 30 minutes, it quadruples every hour. Since 4 hours elapsed from 8:00 A.M. to noon, the number of cells quadrupled 4 times. Therefore, at noon, the number of cells was 60 × 44.

19. A. 123    To find the average speed, in miles per hour, divide the distance, in miles, by the time, in hours. Ali drove 90 miles (123 of 135) in 2.25 hours (135 minutes = 2 hours and 15 minutes = 2123 hours). Then 90 ÷ 2.25 = 40

**Remember that 60 miles per hour is 1 mile per minute. Since Ali drove for 135 minutes and covered less than 135 miles, she was going slower than 60 mph. Eliminate C, D, and E, and guess.

20. C. 123    Since the average of the five students’ weights is 150.4 pounds, their total weight is 5 × 150.4 = 752 pounds (TACTIC E1). No student weighs less than 130 and none is within 5 pounds of another, so the least that the four lightest students can weigh is 130, 135, 140, and 145 pounds, for a total of 550 pounds. The heaviest student, therefore, cannot exceed 752 – 550 = 202 pounds.

Section 4 Writing Skills

  1. D.    Choice D corrects the double negative found in the other four choices. Note that, grammatically, hardly is considered a negative word. Choice C, in addition, creates a run-on sentence.

  2. E.    Comma splice. Choice B is a loose, ineffective compound sentence. Choice C is a comma splice, Choice D is a sentence fragment. Choice E is the most effective sentence; it corrects the comma splice without introducing any fresh errors.

  3. B.    Error in parallelism. In Choice A, the conjunction but should be followed by a clause to parallel the clause in the first half of the sentence.

Choice B provides such a clause. The awkward placement of the word invariably in Choice C makes the sentence very unclear. The use of who in Choices D and E leads to ambiguity because it may be taken to refer to workers.

  4. E.    Error in modification and word order. Choices A, B, and C are incorrect because of the dangling participle considering. Choice D is unidiomatic.

  5. C.    Error in modification and word order. In Choices A, B, and D, the modifier familiar is dangling.

The wording in Choice E suggests that the explorer was a success, whereas the original sentence states that the search was a success—a somewhat different meaning. Choice C corrects the error and retains the original meaning of the sentence.

  6. B.    Error in parallelism. Choices A, C, and E do not maintain parallel structure. Choice B corrects this weakness. The infinitive to cut cannot be an object of demand, as in Choice D; a noun clause like the one in Choice B corrects this error.

  7. A.    Choice A is correct because the subject of the verb had must be who, not whomWhich in Choice E should not be used to refer to a person.

  8. E.    Error in pronoun-number agreement. The pronouns you and your in the second clause of the sentence refer to technician, which is a third person singular noun. The pronoun, therefore, should be the third person singular her.

  9. C.    Choice C corrects the ungrammatical construction by linking two independent clauses with a semicolon.

10. E.    Choice E eliminates the wordiness of the original sentence. It is both clear and effective.

11. A.    Sentence is correct.

12. B.    Adjective and adverb confusion. Change increasing to increasingly.

13. A.    Misuse of relative pronoun. Change who (refers to people) to that (refers to things).

14. B.    Error in pronoun case. Change he to him.

15. B.    Error in diction. Incorrect introduction to noun clause. Change where to in which to modify explosion.

16. C.    Error in parallelism. Change called to were calling.

17. C.    Error in coordination and subordination. Incorrect coordinating conjunction. Change and to but.

18. C.    Error in pronoun number agreement. Change their to its.

19. B.    Error in subject-verb agreement. Change are finding to is finding.

20. B.    Error in tense and shift in pronoun person. Change one finds to you will find.

21. C.    Error in tense. Change have not had to will not have.

22. D.    Error in parallelism. Change to guarantee to guaranteeing.

23. E.    Sentence is correct.

24. E.    Sentence is correct.

25. C.    Error in diction. Change due to to as a result of.

26. D.    Error in tense. Change is questioned by to was questioned by.

27. A.    Pronoun error. Change which (refers to things) to who (refers to people).

28. C.    Error in subject-verb agreement. Both teams intend to review the videos.

29. E.    Sentence is correct.

30. C.    Choice A contains an awkwardly expressed clause that begins which caused. Choice B contains a faulty comparison: not as severe that.

Choice C accurately revises the sentence. It is the best answer. Choice D contains an awkwardly expressed clause that begins that banning. Choice E contains faulty diction. The conjunction and is not an effective connecting word in the context.

31. B.    Choice A contains a dangling participle and a weak passive construction. Choice B accurately continues the thought begun in sentence 9. It is the best answer. Choice C contains redundant language: foul and obscene. Choice D contains a weak passive construction (Obscenities are often heard) and is wordy. Choice E lacks a main verb; therefore, it is a sentence fragment.

32. E.    Choices A and B are adequate, but dull, transitional statements. Choice C is a wordier version of A and B. Choice D contains an error in subject-verb agreement; the subject changes is plural, but the verb has is singular. Choice E serves as a good transitional statement that highlights the most important change in music discussed in the essay. It is the best answer.

33. A.    Only Choice A requires development, since no mention is made in the essay of “new categories” of music. All the other choices are factual statements that require no further elaboration.

34. B.    All choices except B contribute to the discussion of changes in musical sounds brought about by technology. Choice B, however, wanders from the topic.

35. D.    Choice A unnecessarily repeats the phrase in the past thirty years and contains an error in subjectverb agreement. Choice B is awkwardly expressed and confusing. Choice C fails to list the changes in music in the proper order. Also, technology and lyrics appear to be a single item.

Choice D succinctly and accurately states the main idea of the essay. It is the best answer.

Choice E, by subordinating the initial clause, gives lyrics in music undeserved importance.

Section 6 Critical Reading

  1. E.    The depletion or exhaustion of our energy sources would lead us to seek alternative sources.

Remember: in double-blank sentences, go through the answer choices, testing the first words in each choice and eliminating those that don’t fit.

Note that you are looking for a word with negative associations. Therefore, you can eliminate any word with positive ones. Choices A, B, and D all have positive associations. Only Choice C or E can be correct. Turning to the second words of these two choices, you can eliminate Choice C: it would make no sense to seek inefficient energy sources.

(Cause and Effect Signal)

  2. C.    Verbose means excessively wordy. The second clause of the sentence provides an example that brings the abstract term verbose to life.

(Examples)

  3. C.    Bugs are abundant or plentiful because they endure. They endure because they have an extremely low extinction or death rate.

(Cause and Effect Pattern)

  4. D.    To the author of this sentence, pre-Spanish art in Mexico is praiseworthy. It is not crude art but strong art. He likes its bold simplifications and asserts they are not the result of the artists’ inability to overcome or conquer the technical difficulties involved.

Note how the second clause of the sentence serves to clarify what the author means by his assertion that pre-Spanish art is not crude.

Be careful with sentences containing negative words and prefixes. The words “not,” “mistaken,” and “inability” all affect the sentence’s meaning.

  5. E.    To pussyfoot is to refrain from committing oneself, to be wary of stating one’s views candidly. People unable or afraid to take forthright stands are, by definition, equivocators.

(Definition Pattern)

  6. D.    The “property” that Douglass’s ex-master sought to reclaim was a personal possession that had been lost to him, namely, his former slave Frederick Douglass.

  7. C.    The abolitionists criticized Douglass for letting his British friends purchase his freedom.

They were clearly negative about the action, considering it politically incorrect.

  8. B.    Passage 1 is clearly expository: it presents information about a historical figure. Passage 2, in contrast, is argumentative: in it, Douglass justifies his actions, giving his grounds for running away.

  9. C.    The enterprise that Douglass kept secret from his former master was his plan to escape from slavery.

10. A.    The opening paragraph, with its talk of clearly marked outlines and contrasts “between suffering and joy,” and the concluding sentence, with its mentions of “violent contrasts” and “the perpetual oscillation between despair and distracted joy,” emphasize the author’s main idea: the Middle Ages were marked by extremes.

Choice B is incorrect. Though the author depicts aspects of the lives of the very rich and the very poor, he does not stress the notion that their styles complemented one another.

Choice C is incorrect. The author’s concern is for the Middle Ages, not for the twentieth century.

Choices D and E are incorrect. They are unsupported by the text.

Remember, when asked to find the main idea, to check the opening and the summary sentences of each paragraph.

11. B.    The cloaking of minor activities (journeys, visits, etc.) with forms (line 9) “raised them to the dignity of a ritual”; in other words, the forms (fixed or formal ways of doing things) made the acts more dignified.

Choices A, C, D, and E are incorrect. They are not supported by the passage.

Remember: when asked about specific details in the passage, spot key words in the question and then scan the passage to find them (or their synonyms). Key Word: formalities.

12. A.    The linking of “formulae” with “ceremonies” (formal series of acts) and “benedictions” (words of blessing) suggests that these for mulae are most likely set forms of words for rituals.

Note how the use of the colon suggests that all three words that follow are examples of “formalities.”

13. E.    Treat this vocabulary-in-context question as if it were a sentence completion exercise. “Every ____ and estate, every rank and profession, was distinguished by its costume.” Which of the answer choices best fills in the blank? Estate (major political or social class), rank (separate class in a social system), and profession (body of people engaged in an occupation) are all examples of groups or classes of people.

Thus, in this context, an order is a social class.

Note how the use of “and” and of the commas to group together these terms suggests that all four are similar in meaning.

14. A.    The last sentence of the third paragraph states that the lofty churches, the houses of worship, towered above the town. The churches always “remained dominant.”

When asked about specific details, spot the key words in the question and scan the passage to find them (or their variants).

Key Words: above, towered.

15. A.    In cataloging the extremes of medieval life, the author in no way suggests that the Middle Ages were boring.

Choice B is incorrect. The author portrays the Middle Ages as festive and joyful; he says they were filled with vivid pleasures and proud celebrations.

Choice C is incorrect. The author portrays the Middle Ages as filled with ceremony and ritual.

Choice D is incorrect. The author portrays the Middle Ages as passionate and turbulent; he mentions the “tone of excitement and of passion” in everyday life.

Choice E is incorrect. The author suggests the Middle Ages were harsh and bleak; he portrays them as cold and miserable.

16. D.    Richard’s introduction to school, his parents’ reaction to their unfriendly neighbors, his brother and sister’s silence about their classroom experiences—all these instances illustrate the family members’ interaction with the English-speaking world.

17. D.    The older children return home speaking Spanish, abandoning the English taught in the classroom. What is more, “their mysterious books . . . remained on the table next to the door, closed firmly behind them.” Clearly, they readily ignored the need to practice using English at home.

18. B.    The author’s statement that it “was the first time I had heard anyone name me in English” supports Choice B. In addition to finding himself apart from his family, the usual experience of new pupils, he finds himself stripped of his name, his identity. Being addressed in such a strange and impersonal manner rattles him.

Choice A is incorrect. All the students were uneasy to find themselves separated from their families.

Choices C and D are incorrect. Nothing in the passage supports them.

Choice E is incorrect. The narrator lived in a gringo neighborhood; he must have seen white children.

19. C.    Bilingual education is a scheme or plan that seeks to permit non-English-speaking children to use their native languages in school.

Choice E is incorrect. As used in the sentence, bilingual education is a plan or program.

Establishing such a plan is the goal of bilingual education’s supporters.

20. B.    Bilingual education’s supporters wish to have non-English-speaking children taught in the language they customarily use at home.

Rodriguez feels they have missed an important point. To him, the job of public education is to teach children to function effectively in the society in which they live. To do so, they must learn to use public language, the language used outside the home. He believes that the champions of bilingual education fail to see that public education must meet public needs, not necessarily personal ones.

21. C.    Rodriguez has just given his opinion on a controversial topic. He now must convince his readers that he knows what he is talking about. To do so, he cites specific aspects of his background that prove he knows something about bilingual education. In other words, he indicates what qualifies him to speak authoritatively on the issue.

22. A.    The author’s assertions in the last paragraph that his parents coped very well and that they were nobody’s victims, indicate that his basic attitude toward them is admiring.

23. B.    Statement B is true to the author’s experience: they felt estranged from the gringos’ world.

Choice A is incorrect. Richard’s father found steady work.

Choice C is incorrect. Although Sacramento as a whole was not determined to keep Mexicans out of white neighborhoods, some neighbors tried to frighten away Richard’s family.

Choice D is incorrect. Lacking confidence in public, Richard’s parents remained detached from community affairs.

Choice E is incorrect. Richard’s parents sent their children to Roman Catholic schools; they were involved with academic institutions.

24. E.    For Richard’s parents to call white people los americanos, “the Americans,” implies that on some level they did not consider themselves Americans. This is a telling or revealing comment that points up the degree of alienation Richard’s parents felt.

Section 7 Mathematical Reasoning

MULTIPLE-CHOICE QUESTIONS

  1. C.    The product of any set of numbers that includes 0 is 0, and the absolute value of 0 is 0.

  2. D. 123    Let the number be x, and write the equation: 123 Multiply both sides by 12: 9x = 84 + 2x. Subtract 2x from each side and divide by 7: 123

Be careful: 12 is not the answer. You were asked for 123 of the number: 123(12) = 20.

  3. B.    A number that is a member of both A and B must be an integer that is a multiple of both 5 and 7, and so must be a multiple of 35. There are 5 multiples of 35 less than 200: 35, 70, 105, 140, 175.

  4. C.    123 Cube both sides:

x + 1 = 23 = 8 123 x = 7.

**Test the choices, starting with C. If x = 7,

image

We got lucky; choice C works.

  5. B.    Write the equation and use the six-step method (TACTIC G1) to solve it:

123

Plugging in 5 for x, we get the lengths of the sides: 8, 7, and 10.

  6. D.    A circle can cross each side of a square at most twice.

The answer is 2 × 4 = 8.

123

  7. C.    AC is a diagonal of square ABCD. By the distance formula 123 123

By KEY FACT K8, one formula for the area of a square is 123, where d is the length of the diagonal. Then the area of ABCD is 123 = 123 = 17.

**If you don’t know the formula image, then once you know that the length of diagonal AC is image, you can find s, the length of a side, by dividing by image, and then A = s2 = 17.

  8. A.    If f(x) = x + 5, then f(3a) = 3a + 5 and

f(2a) = 2a + 5. Therefore,

f(3a) + 2 = f(2a) + 3 123

3a + 5 + 2 = 2a + 5 + 3 123

3a + 7 = 2a + 8 123 a = 1.

**Use TACTIC 5: test the answer choices. When

a = 1,

f (3a) + 2 = f (3) + 2 = (3 + 5) + 2 = 10

and

f (2a) + 3 = f (2) + 3 = (2 + 5) + 3 = 10.

GRID-IN QUESTIONS

  9. (13) 123    Use your calculator:

123

10. (2.5 or 123123    Replace a by 44 and b by 275:

123

123

Then, 123

11. (150)    Draw and label the right triangle. By the Pythagorean theorem: 152 + b2 = 252 123 225 + b2 = 625 123 b2 = 400 123 b = 20. The area of the triangle is 123(15)(20) = 150.

123

[You can save some work if you recognize this as a 3-4-5 triangle in which each side has been multiplied by 5 (15-20-25).]

12. (87.5) 123    If the average of 5 numbers (a, b, c, d, e) is 95, the sum of these numbers is 5 × 95 = 475 (TACTIC E1). Similarly, the sum of the 3 numbers a, b, and e whose average is 100 is 300, leaving 175 (475 – 300) as the sum of the 2 remaining numbers, c and d. The average of c and d is their sum divided by 2: 175 ÷ 2 = 87.5 .

13. (110) 123 Here, x2 – y2 = (x + y)(x – y) = 10 × 11 = 110.

**Adding the two given equations, you get 2x = 21, so x = 10.5. Since 10.5 + y = 10, then y = –0.5.

Evaluate using your calculator:

x2 – y2 = (10.5)2 – (0.5)2 = 110.25 – 0.25 = 110.

14. (52) Ignore the x’s and the y’s. In any “staircase” the perimeter is just twice the sum of the height and the length, so the perimeter is 2(12 + 14) = 2(26) = 52.

15. 123

123 123

**Use TACTIC 6. If we let 123 That’s OK, but it’s easier if we avoid fractions.

We’ll let d = 5. Then, 123 and a = 2(8) = 16, so = 123

16. (125) 123 Since the diameters of the tables are in the ratio of 36:24, or 3:2, the ratio of their areas is 32:22 = 9:4. Convert the ratio to a percent: 9:4 = image = 2.25 = 225%. The area of the larger table is 225% of the area of the small one, or is 125% more than the area of the small one.

123 **With your calculator, actually calculate the areas. The radius of the larger table is 18, so its area is π(18)2 = 324π. Similarly, the radius of the smaller table is 12, and its area is π(12)2 = 144π. The difference in the areas is 324π − 144π = 180π, and 180π is 125% of 144π:

123

17. (12) Let x = Adam’s age in 1980. Then, in 1980, Judy’s age was 3x and Elaine’s age was 8x. Since Elaine is 20 years older than Judy, 8x = 3x + 20 ⇒ 5x = 20 ⇒ x = 4.

Therefore, in 1988, Adam was 4 + 8 = 12.

**Test numbers for Adam’s age in 1980 and zoom in.

image

18. (490) 123 Let x be the number Jessica chose. Then the other terms are as follows:

123

Finally, 4x + 36 = 1996 123 4x = 1960 123 x = 490.

**Work backwards. Treat 1996 as the first term and alternately take half of the previous term and subtract 6 from the pevious term:

1996 123 998 123 992 123 496 123 490

Section 8 Critical Reading

  1. C.    The publisher is apologizing for having used the photographs without the photographer’s permission. He regrets printing them without authorization. To answer this question correctly, you must check the second word of each answer choice.

(Definition Pattern)

  2. B.    To acquire a taste for something, you must originally not have that taste or even dislike the item; you acquire the taste by growing to like or welcome it.

Note how the second clause of the sentence serves to clarify what is meant by the term “acquired taste.”

(Example)

  3. D.    Assiduous work, work performed industriously or diligently, should not lead employers to complain.

Note that the use of because in the opening clause signals that a cause and effect relationship is at work here.

(Cause and Effect Signal)

  4. B.    The key word here is obstacle. What sort of attitude toward modern art would present an obstacle or hindrance to artists? Clearly, a negative one. You therefore can eliminate any answer choice that is positive. Veneration(awed respect) is highly positive. Eliminate Choice A.

Indifference (lack of caring), disdain (scorn), ignorance, and intolerance are all negative terms. You must check the second word of Choices B, C, D, and E. Bacon and Freud are now accepted, respected artists. In years past, they were viewed differently. They were dismissed or rejected as insignificant. The correct answer is Choice B.

Note that a secondary meaning of dismissed is involved here.

(Contrast Pattern)

  5. D.    By definition, volatile substances tend to evaporate (convert from a liquid state into a vapor).

Beware of Eye-Catchers. Choice A is incorrect.

Insoluble substances cannot be dissolved in liquid. Such substances are unlikely to evaporate.

(Definition)

  6. B.    By showing that they had no hidden or concealed weapons, they were showing themselves to be friendly or amicably disposed.

Because the first word of any one of these answer choices could work, you have to try out each entire pair before eliminating any of the choices.

(Cause and Effect Signal)

  7. B.    Auden finds it curious or unusual that detective fiction most appeals to people who are least likely to find other forms of escapist literature appealing.

  8. C.    In lines 1–4, Auden states that typical readers of detective fiction do not find other forms of daydream or escapist literature appealing. In that context he then states that the typical mystery fan could not stomach or tolerate True Confessions and movie magazines. This suggests that Auden considers these magazines standard forms of escapist literature.

  9. D.    In the closing paragraph of Passage 1, Auden states that readers of detective fiction indulge in a fantasy of escape or release that is prompted by a “feeling of guilt, the cause of which is unknown to the dreamer.” Thus, they are seeking momentary release from a vague sense of guilt.

Choice A is incorrect. Nothing in the passage supports it.

Choice B is incorrect. Auden denies that readers of detective fiction are bent on satisfying “violent or murderous wishes.”

Choice C is incorrect. Although Auden depicts readers of detective fiction as dreamers, he depicts them as dreamers impelled by a sense of guilt, not by a sense of boredom.

Choice E is incorrect. Nothing in the passage supports it.

10. E.    To translate murderous wishes into action is to convert or switch from dreaming about murder to committing the actual crime.

11. B.    Kafka’s The Trial is cited as an “instructive example of the difference between a work of art and the detective story.” Auden then goes on to analyze The Trial to point out its qualities as a work of art that distinguish it from mere detective fiction.

Choice A is incorrect. The Trial is a work of art, not a detective story.

Choice C is incorrect. Auden is not discussing readers of detective fiction in lines 46–54.

Choice D is incorrect. The outside investigator, the genius who removes guilt by giving knowledge of guilt, is a figure out of the detective story; he has no place in the work of art.

Although K investigates his situation, he is trapped inside it; he is no genius from outside.

Choice E is incorrect. There is nothing in the passage to support it.

12. E.    Auden explicitly disassociates himself from the readers of thrillers (which he rarely enjoys). However, he associates himself with the readers of detective fiction (“me and my neighbors”), those who are caught up in the mystery, but, unlike the outside investigator, unable to solve it. This suggests he is a fan of detective fiction, one who views it with genuine appreciation.

Choices A, B, C, and D are incorrect. Nothing in the passage suggests them.

13. C.    In the phrase “use up our intellectual shoe leather,” the author of Passage 2 evokes a familiar image of the detective wearing out his or her shoe leather while pounding the pavements in search of clues. Readers of mysteries do not physically pound the pavements searching for clues. However, they do work hard mentally, much as detectives do physically.

14. D.    The author writes of taking “pleasure in the discovery . . . that there is more work to be done.” This suggests that he finds the prospect of additional work pleasing or delightful.

15. B.    Throughout Passage 2 the author is discussing detective fiction. Immediately before mentioning Marlowe and Archer and their search for the facts, the author refers to the objective truth defined or discovered by the researcher/detective. This juxtaposition suggests that Marlowe and Archer are fictional detectives.

16. B.    To employ a word is to use it.

17. E.    The statement that the detective story begins “in search of the subject . . .(and) asking, What is the question?” suggests that the story must start by defining the problem that is to be solved in the course of the investigation.

18. B.    In Passage 1, Auden goes on at some length about the psychological satisfaction readers of detective fiction derive from their literary “escape” from their sense of guilt. In Passage 2, Winks describes (also at some length) the pleasure readers of detective fiction get from raising questions, pursuing truths, making judgments. In both passages, the authors are primarily concerned with the question of why people enjoy reading detective fiction.

19. A.    In characterizing detective fiction as “a bit like a religion” and linking it “with all fine literature, history, philosophy,” Winks clearly goes further than Auden would. Auden stresses that detective stories are “escape literature, not works of art” (lines 19 and 20). Thus, Auden would most likely react to Winks’s somewhat exalted view of detective fiction by reiterating that reading detective fiction is an escape, not a highly serious pursuit.

Section 9 Mathematical Reasoning

  1. B.    Multiplying both sides of 2x – 1 = 9 by 5 yields 10x – 5 = 45.

**Just solve: 2x – 1 = 9 123 2x = 10 123 x = 5 123 10x = 50 123 10x – 5 = 45.

  2. E.    Since the measures of corresponding angles are equal, a = b (KEY FACT I6). Since the figure is not drawn to scale, the angles could just as well be acute as obtuse, as shown in the figure below. The sum a + b cannot be determined from the information given.

123

  3. D.    Since there are no parentheses, you must be careful to follow the proper order of operations (PEMDAS). Do multiplications and divisions left to right before any additions and subtractions.

A: 4 × 4 ÷ 4 + 4 = 16 ÷ 4 + 4 = 4 + 4 = 8

B: 4 ÷ 4 × 4 + 4 = 1 × 4 + 4 = 4 + 4 = 8

C: 4 × 4 – 4 × 4 = 16 – 16 = 0

D: 4 ÷ 4 + 4 × 4 = 1 + 16 = 17, the greatest value

E: 4 + 4 × 4 – 4 = 4 + 16 – 4 = 20 – 4 = 16

  4. C. 123    Let 4x and 3x be the numbers of students taking and not taking Spanish, respectively. Then 4x + 3x = 840 123 7x = 840 123 x = 120. The number taking Spanish is 4(120) = 480.

**Use TACTIC 5. Try choice C. If 480 students take Spanish, 840 – 480 = 360 do not.

Is 123 a true proportion?

Yes. Cross-multiply: 480 × 3 = 360 × 4.

  5. C.    The sum of three consecutive integers can be expressed as

n + (n + 1) + (n + 2) = 3n + 3 = 3(n + 1), and so must be a multiple of 3. Only 28 is not a multiple of 3.

**Quickly add up sets of three consecutive integers: 4 + 5 + 6 = 15, 5 + 6 + 7 = 18, 6 + 7 + 8 = 21, and so on, and see the pattern (they’re all multiples of 3); or cross off the choices as you come to them.

  6. B.    By KEY FACT A21, 123 and 123 123

So, n=4.

  7. E.    Since k || 123, in the figure below, by KEY FACT I6, z + (3x + 15) = 180. But z = x + 5 (KEY FACT I4: vertical angles are congruent).

123

Then, (3x + 15) + (x + 5) = 180 123 4x + 20 = 180 123 4x = 160 123 x = 40. So z = x + 5 = 45. Also, w + (2x + 30) = 180. But 2x + 30 = 80 + 30 = 110, so w = 70. Finally, w + y + z = 180 123 70 + y + 45 = 180. So, 115 + y = 180 and y = 65.

  8. D.    This is a repeating sequence with 3 terms (1, 2, 3) in the set that repeats. Since the sum of the 3 numbers in each set is 6, the sum of the first 33 sets or 99 terms is 6 × 33 = 198.

Since the next term is 1, the sum of the first 100 terms is 199.

  9. C.    Check each of the three statements. Since the average of a and a is a, I is true. Clearly, the average of a and b is the same as the average of b and a, so II is also true. Note that a 123 (123 c) is not the average of the three numbers a, b, and c. To calculate this average, you first take the average of b and c and then take the average of that result and a.

Then,

123,

whereas

123

and these are equal only if a = c. Therefore, III is false and I and II only are true.

**Use TACTIC 6: plug in some easy numbers. You probably don’t need to do this for I and II, but for III it may be a lot easier than the analysis above:

123

whereas

123

10. B. 123    As always, with a percent problem use a simple number such as 10 or 100. Assume that model A sells for $10; then, since 60% of 10 is 6, model B sells for $16. The chart tells you that 9000 model A’s and 10,000 model B’s were sold, for a total of $10(9000) + $16(10,000) = $90,000 + $160,000 = $250,000. The sales of model A ($90,000) represent 36% of the total sales ($250,000).

11. D.    Use TACTIC 7: choose a number. Let BE, the length of a side of the smallest square, be 1. Then DH = 2 and AB = 4; the areas of the three squares are 1, 4, and 16, respectively. Therefore, the shaded area is 16 – 4 – 1 = 11, and the probability that a point chosen at random inside ABCD is in that shaded region is 123

12. D.    The best approach is to use the six-step method from Section 12-G.

To get rid of the fractions, multiply both sides of the equation by 3: 3x = 2(x + y)

Use the distributive law to

get rid of the parentheses:     3x = 2x + 2y

Subtract 2x from each side:     x = 2y

13. D.    Draw in radii OA and OB. Then, by KEY FACT L10, OA 123 PA and OB 123 PB, so quadrilateral OAPB has two right angles and a 50° angle. Since the sum of all four angles is

360°, you have

90 + 90 + 50 + m 123AOB = 360 123

230 + m 123AOB = 360, and so m 123AOB = 130.

14. A.    The equation of the line can be written in the form y = mx + b, where b is the y-intercept and m is the slope. Since the line crosses the y-axis at 2, b = 2 and the answer must be A or B. Since the line has a negative slope, the 1answer must be A, y = – 123 x + 2.

**Since the line passes through (0, 2) and (4, 0), its slope 123

15. E.    If PQ is a diameter of the circle, then the radius is 1 and A, the area, is 123. This is the smallest possible value of A, but A can actually be any number larger than 123 if the radius is made arbitrarily large, as shown by the figures below.

123

The answer is A 123 π.

16. E.    The easiest way to solve this is to use TACTIC 6. Let x = 2 and y = 1. Then xy = 2, a = 4, and b = 0. Now, plug in 4 for a and 0 for b, and see which of the five choices is equal to 2. Only E works:

123

**Here is the correct algebraic solution.

123

This is the type of algebra you want to avoid.

Section 10 Writing Skills

  1. D.    Wordiness. Choice D makes the writer’s point simply and concisely.

  2. A.    Sentence is correct. Remember: the subject’s grammatical number is not changed by the addition of a phrase that begins with along with, together with, or a similar expression. The subject, coach, is singular. The verb should be singular as well.

  3. E.    Run-on sentence. Choice E eliminates the original comma splice to produce a balanced sentence.

  4. C.    Error in logical comparison. Compare veterans with veterans, not veterans with wars.

  5. C.    Misplaced modifier. Who are acting suspiciously? Not the bags, but the people who packed them!

  6. A.    Sentence is correct.

  7. D.    Error in logical comparison. Compare prices with prices, not prices with cities.

  8. C.    Error in pronoun-antecedent agreement. The subject of the sentence is inoculations (plural). The pronoun should be plural as well. In this particular instance, the plural pronoun they has been replaced by the noun phrase such inoculations.

  9. A.    Sentence is correct.

10. C.    Error in subordination. The use of the conjunction Although in Choice C signals the contrast between what one might have expected (i.e., that the prospectors, who arrived in 1848, would become known as the forty-eighters) and what actually took place (they became known as the forty-niners).

11. D.    Misplaced appositional phrase. Who was once a leading light of the Harlem Renaissance?

Clearly, Hurston was. Choice D correctly positions the word being described (Hurston) closer to the descriptive phrase.

12. A.    Sentence is correct.

13. C.    Error in logical comparison. Compare a percentage with another percentage, not a percentage with a period of time.

14. E.    Double negative. The simple change from hardly no to hardly any corrects the error without introducing new ones.